Решение уравнений с матрицами: Обратная матрица онлайн

Содержание

Решение матричных уравнений — Журнал «Код»

Линейная алгебра и, в частности, матрицы — это основа математики нейросетей. Когда говорят «машинное обучение», на самом деле говорят «перемножение матриц», «решение матричных уравнений» и «поиск коэффициентов в матричных уравнениях». 

Понятно, что между простой матрицей в линейной алгебре и нейросетью, которая генерирует котов, много слоёв усложнений, дополнительной логики, обучения и т. д. Но здесь мы говорим именно о фундаменте. Цель — чтобы стало понятно, из чего оно сделано. 

Краткое содержание прошлых частей: 

  • Линейная алгебра изучает векторы, матрицы и другие понятия, которые относятся к упорядоченным наборам данных. Линейной алгебре интересно, как можно трансформировать эти упорядоченные данные, складывать и умножать, всячески обсчитывать и находить в них закономерности. 
  • Вектор — это набор упорядоченных данных в одном измерении. Можно упрощённо сказать, что это последовательность чисел. 
  • Матрица — это тоже набор упорядоченных данных, только уже не в одном измерении, а в двух (или даже больше).  
  • Матрицу можно представить как упорядоченную сумку с данными. И с этой сумкой как с единым целым можно совершать какие-то действия. Например, делить, умножать, менять знаки.
  • Матрицы можно складывать и умножать на другие матрицы. Это как взять две сумки с данными и получить третью сумку, тоже с данными, только теперь какими-то новыми. 
  • Матрицы перемножаются по довольно замороченному алгоритму. Арифметика простая, а порядок перемножения довольно запутанный. 

И вот наконец мы здесь: если мы можем перемножать матрицы, то мы можем и решить матричное уравнение.

❌ Никакого практического применения следующего материала в народном хозяйстве вы не увидите. Это чистая алгебра в несколько упрощённом виде. Отсюда до практики далёкий путь, поэтому, если нужно что-то практическое, — посмотрите, как мы генерим Чехова на цепях Маркова.

Что такое матричное уравнение

Матричное уравнение — это когда мы умножаем известную матрицу на матрицу Х и получаем новую матрицу. Наша задача — найти неизвестную матрицу Х.

9 простых задач на математику

Шаг 1. Упрощаем уравнение 

Вместо известных числовых матриц вводим в уравнение буквы: первую матрицу обозначаем буквой A, вторую — буквой B. Неизвестную матрицу X оставляем. Это упрощение поможет составить формулу и выразить X через известную матрицу.

Приводим матричное уравнение к упрощённому виду

Шаг 2. Вводим единичную

матрицу 

В линейной алгебре есть два вспомогательных понятия: обратная матрица и единичная матрица. Единичная матрица состоит из нулей, а по диагонали у неё единицы. Обратная матрица — это такая, которая при умножении на исходную даёт единичную матрицу. 

Можно представить, что есть число 100 — это «сто в первой степени», 1001

И есть число 0,01 — это «сто в минус первой степени», 100-1

При перемножении этих двух чисел получится единица:
1001 × 100-1 = 100 × 0,01 = 1.  

Вот такое, только в мире матриц. 

Зная свойства единичных и обратных матриц, делаем алгебраическое колдунство. Умножаем обе известные матрицы на обратную матрицу А-1. Неизвестную матрицу Х оставляем без изменений и переписываем уравнение: 

А-1 × А × Х = А-1 × В  

Добавляем единичную матрицу и упрощаем запись: 

А-1 × А = E — единичная матрица 

E × Х = А-1 × В — единичная матрица, умноженная на исходную матрицу, даёт исходную матрицу. Единичную матрицу убираем

Х = А-1 × В — новая запись уравнения 

После введения единичной матрицы мы нашли способ выражения неизвестной матрицы X через известные матрицы A и B. 

💡 Смотрите, что произошло: раньше нам нужно было найти неизвестную матрицу. А теперь мы точно знаем, как её найти: нужно рассчитать обратную матрицу A-1 и умножить её на известную матрицу B. И то и другое — замороченные процедуры, но с точки зрения арифметики — просто.  

Шаг 3. Находим обратную матрицу

Вспоминаем формулу и порядок расчёта обратной матрицы: 

  1. Делим единицу на определитель матрицы A. 
  2. Считаем транспонированную матрицу алгебраических дополнений. 
  3. Перемножаем значения и получаем нужную матрицу.
Формула вычисления обратной матрицыПервое действие. Мы посчитали определитель и убедились, что он не равен нулю, — это значит, что у матричного уравнения есть вариант решения и можно продолжатьВторое действие, часть 1: получаем матрицу миноровВторое действие, часть 2: переводим матрицу миноров в транспонированную матрицу алгебраических дополнений

Собираем формулу и получаем обратную матрицу. Для удобства умышленно оставляем перед матрицей дробное число, чтобы было проще считать.

Третье действие: получаем обратную матрицу

Шаг 4. Вычисляем неизвестную матрицу

Нам остаётся посчитать матрицу X: умножаем обратную матрицу А-1 на матрицу B. Дробь держим за скобками и вносим в матрицу только при условии, что элементы новой матрицы будут кратны десяти — их можно умножить на дробь и получить целое число. Если кратных элементов не будет — дробь оставим за скобками.

Решаем матричное уравнение и находим неизвестную матрицу X. Мы получили кратные числа и внесли дробь в матрицу

Шаг 5. Проверяем уравнение

Мы решили матричное уравнение и получили красивый ответ с целыми числами. Выглядит правильно, но в случае с матрицами этого недостаточно. Чтобы проверить ответ, нам нужно вернуться к условию и умножить исходную матрицу A на матрицу X. В результате должна появиться матрица B. Если расчёты совпадут — мы всё сделали правильно. Если будут отличия — придётся решать заново. 

👉 Часто начинающие математики пренебрегают финальной проверкой и считают её лишней тратой времени. Сегодня мы разобрали простое уравнение с двумя квадратными матрицами с четырьмя элементами в каждой. Когда элементов будет больше, в них легко запутаться и допустить ошибку.

Проверяем ответ и получаем матрицу B — наши расчёты верны

Ну и что

Алгоритм решения матричных уравнений несложный, если знать отдельные его компоненты. Дальше на основе этих компонентов математики переходят в более сложные пространства: работают с многомерными матрицами, решают более сложные уравнения, постепенно выходят на всё более и более абстрактные уровни. И дальше, в конце пути, появляется датасет из миллионов котиков. Этот датасет раскладывается на пиксели, каждый пиксель оцифровывается, цифры подставляются в матрицы, и уже огромный алгоритм в автоматическом режиме генерирует изображение нейрокотика:

Этого котика не существует, а матрицы — существуют. 

Текст:

Александр Бабаскин

Редактура:

Максим Ильяхов

Художник:

Даня Берковский

Корректор:

Ирина Михеева

Вёрстка:

Мария Дронова

Соцсети:

Олег Вешкурцев

что нужно знать для их решения OTUS

Матрица – это своеобразный математический объект, который записывается в виде таблицы элементов. Обычно представлен прямоугольником или квадратом. Математический объект, который записывается в виде таблицы компонентов, состоящей из определенного количества строк и столбцов.

С соответствующими составляющими можно выполнять разные действия. Пример – решать уравнения. Именно об этом пойдет речь далее. Информация пригодится как математикам, так и сотрудникам IT-сферы.

Матричное уравнение – это…

Матричное уравнение – это уравнение, которое напоминает линейный (числовой) аналог. Но в качестве элементов в нем используются матрицы.

Типовое уравнение подобного характера включает себя ранее упомянутые математические объекты, а также некоторую неизвестную матрицу X. Именно ее и необходимо вычислить.

Что потребуется для обнаружения результата

Для того, чтобы найти значение неизвестного, которое содержит уравнение, каждый должен сначала тщательно изучить теорию. Без этого проводить необходимые манипуляции не получится.

Решить матричное уравнение можно, если заучена следующая теория:

  • понятие соответствующего объекта и его составляющих;
  • определитель матрицы;
  • ключевые операции над рассматриваемыми компонентами;
  • определение столбцов матрицы, главной диагонали;
  • что такое транспонированная матрица.

Также человек должен уметь решать линейные числовые уравнения. Без всего этого смысл изначально поставленной задачи теряется.

Сложение и вычитание

Для того, чтобы должным образом можно было решить пример, содержащий матричные компоненты, необходимо хорошо разбираться в элементарных операциях с ними. Первый вариант – сложение и вычитание.

Стоит запомнить – решать задачи по сложению удастся лишь матрицы одного и того же размера. Результатом станет математический объект аналогичного «объема».

Сам процесс вычислений достаточно прост – нужно сложить или вычесть соответствующие элементы в столбцах и строках.

Выше – наглядный пример сложения. Вычитание производится аналогичным образом.

Умножение на число

Матрицы можно умножать на то или иное число. Такая манипуляция с легкостью производится любым математиком. Решение задачи напоминает линейные примеры.

Здесь необходимо запомнить следующие данные:

  • операции подобного плана возможны с матрицами любого размера;
  • для получения результата на необходимое число нужно умножить каждый элемент упомянутого мат объекта;
  • полученный результат – матричный компонент аналогичного размера.

Выше – наглядный пример того, как осуществляется умножение на число 5.

Перемножение друг с другом

Умножение матриц между собой – манипуляция, которую можно осуществлять не всегда. Пример – даны мат объекты A и B. Их удастся перемножить, если количество столбцов матрицы a будет равно строкам матрицы b.

Каждый компонент, получившийся в ходе расчетов, стоящий в i-строке и j-столбце равен сумме произведений соответствующих элементов в i-строчке первого множителя и j-столбце второго.

Данный шаблон и наглядный пример помогут найти грамотное решение при перемножении матричных объектов.

Транспонирование

Математика – наука, в которой формулы матриц и иных составляющих играют важную роль. Если не усвоена теория, решить поставленную задачу не получится. Для уравнений может потребоваться транспонированная ма трица.

При транспонировании строки и столбцы будут меняться местами. На письме такие компоненты обознаются как AT.

Это – наглядный пример того, как осуществляется соответствующая математическая операция.

Определитель

В случае с определителем необходимо уяснить следующие данные:

  • определитель – это детерминант;
  • представляет собой численную характеристику квадратной матрицы;
  • при помощи определителя удастся решить множество математических задач;
  • для нахождения соответствующего результата необходимо вычислить разность произведение составляющих главной и побочной диагоналей.

Определитель матрицы первого порядка (это – единичные матрицы) будет равен этому самому компоненту. Если речь идет об объекте размером 3×3, ситуация усложняется.

Для нее значение детерминанта будет равно сумме произведений компонентов главной диагонали и произведений составляющих на треугольниках с гранью параллельной главной. От последней нужно отнять произведение элементов побочной диагонали и произведение чисел, лежащих на треугольниках с гранью параллельной побочной диагонали.

На практике вычисление детерминантов больших размеров встречается редко. Некоторые, чтобы не запутаться, для решения поставленной задачи пользуются разнообразными онлайн калькуляторами. Их смысл – объяснение вычислительного процесса, а также выдача грамотных результатов.

Обратная матрица

Для каждого числа a, которое не равно нулю, существует обратное a-1. Оно будет таким, что произведение оных на выходе даст единицу. Формула записи проста: a*a-1=1. Это понятие подходит и для квадратных матриц.

Матрица A-1 будет обратной по отношению к матрице A, если при умножении оной на данную, как справа, так и слева, получится единичная матрица. К таковым относят математические объекты, включающие в себя всего один элемент (то есть, одну строку и столбец).

Не каждый квадратный матричный элемент имеет обратную матрицу. Если a не равно нулю – это достаточное и необходимое условие существования a-1, то для существования A-1 соответствующим требованием будет |A| не равен нулю.

Миноры и дополнения

Для полного понимания теории, связанной с матрицами и уравнениями, нужно разобраться с понятием алгебраических дополнений, а также минором:

  • Если в определителе n-го порядка происходит вычеркивание i-строки и k-столбца, на пересечении которых расположен объект aik, то полученный детерминант (n-1)-порядка будет обозначаться минором Mik.
  • Минор с определенным знаком, который находится в зависимости от четности суммы i+k номеров строчки и столбца, на пересечении которых расположен компонент aik, — это алгебраическое дополнение. Обозначается как Aik=(-1)i+kMik.
  • Когда в детерминанте порядка n все составляющие последней строчки (столбца), за исключением компонента, стоящего в правом нижнем углу, равняются нулю, то определитель – это произведение соответствующего элемента на минор.
  • Если у детерминанта все составляющие строчки/столбца за исключением одного равняются 0, то определитель – это произведение этого самого компонента на алгебраическое дополнение.

Наглядные примеры и доказательства перечисленных утверждений, раскрывающие их смысл, можно обнаружить по этой ссылке.

Уравнения и их решение

Теперь, когда уделено время произведению матрицы на число, а также иным элементарным операциям с рассматриваемыми объектами математики, можно приступать к непосредственному решению уравнений. Это не самый трудный процесс. Математика здесь находится примерно на школьном уровне.

Формула вычислений у матричных уравнений – точно такая же, как и простых алгебраических, в которых есть умножение. Здесь на помощь придет теория относительно обнаружения произведения матриц.

Пусть будут даны: A * X = B или X * A = B, где A и B – это известные матрицы, а X – неизвестная. Далее ситуация будет зависеть от конкретных обстоятельств:

  1. В первом случае, когда речь идет об уравнении A * X = B, обе части требуется умножить на обратную к A матрицу A-1 с левой стороны. Получится E * X = A-1 * B, где E – единичная матрица. Отсюда следует, что E * X = X. Результат вычислений – X = A-1 * B.
  2. Во втором случае, при уравнении X * A = B ситуация будет обстоять аналогичным образом. Но направление умножения на матрицу, обратную матрице A, меняется. Элемент B будет перемножаться с ней с правой стороны. Получится X * A * A-1 = B * A-1. Итог – X = B * A-1.
  3. Есть и третий случай – когда неизвестная матрица в уравнении расположена в середине произведения трех матриц: A * X * B = C. Здесь нужно известную матрицу из левой части умножить на обратную той, что стоит слева в заданном уравнении. И справа на матрицу, обратную той, что была с правой стороны. Итог будет следующим: X = A-1 * C * B-1.

Если же X в заданном примере – это обычное число, то формула обнаружения результата будет точно такой же, как и в линейных уравнениях.

Как лучше разобраться в теме

С формулой матрицы, а также ее основными компонентами теперь все понятно. И с основными операциями тоже удалось познакомиться. Отныне с легкостью найдем матрицу даже в уравнении при необходимости.

Для того, чтобы лучше вникнуть в соответствующую тему, стоит хорошенько изучить школьный курс математики, а также алгебру на 1 курсе обучения в ВУЗах. Информация пригодится как ученым, так и программистам.

Научиться коддить можно на специализированных дистанционных курсах. Они помогут быстро вникнуть в основы математики и информатики, а также создания приложений и игр. Курс рассчитан на срок до года. В процессе даже новичок, далекий от точных наук, сможет разобраться с матрицами и коддингом. А еще человек получит бесценную практику и новые полезные знакомства.

Хотите освоить современную IT-специальность? Огромный выбор курсов по востребованным IT-направлениям есть в Otus!

Также, возможно, вам будет интересен следующий курс:

Матричные уравнения с примерами решения

Содержание:

  1. Примеры с решением

Обратной матрицей к квадратной матрице А называется такая матрица (обозначаетсячто ЛЗамечание. Если матрица существует, то она единственна.

Присоединенной матрицей к квадратной матрице называется матрица полученная транспонированием из матрицы, составленной из алгебраических дополнений к элементам Теорема 1.3. Если квадратная матрица А — невырожденная (т. е. ), то (4.1)

Метод присоединенной матрицы вычисления обратной матрицы к невырожденной матрице А состоит в применении формулы (4. 1). Метод элементарных преобразований (метод Гаусса) вычисления обратной матрицы к невырожденной матрице А состоит в следующем.

Приписывая справа к матрице А размера единичную матрицу размера получим прямоугольную матрицу размера С помощью элементарных преобразований над строками матрицы Г сначала приведем ее к ступенчатому виду где матрица — треугольная, а затем к виду Матричные уравнения простейшего вида с неизвестной матрицей записываются следующим образом (4.2) (4.3) (4,4)

По этой ссылке вы найдёте полный курс лекций по высшей математике:

Высшая математика: лекции, формулы, теоремы, примеры задач с решением

В этих уравнениях — матрицы таких размеров, что все используемые операции умножения возможны, и с обеих сторон от знаков равенства находятся матрицы одинаковых размеров. Если в уравнениях (4.2), (4.3) матрица А невырожденная, то их решения записываются следующим образом: X Если в уравнении (4. 4) матрицы А и С невырождены, то его решение записывается так:

В этих уравнениях А, В, С, X — матрицы таких размеров, что все используемые операции умножения возможны, и с обеих сторон от знаков равенства находятся матрицы одинаковых размеров. Если в уравнениях (4.2), (4.3) матрица А невырожденная, то их решения записываются следующим образом: Если в уравнении (4.4) матрицы А и С невырождены, то его решение записывается так:

Примеры с решением

Пример 1.

Найти (методом присоединенной матрицы) матрицу, обратную к данной:

Найдем det А:

Так как det , то матрица существует.

Возможно вам будут полезны данные страницы:

Объём тела вращения

Разложение в ряд Тейлора

Метод Гаусса: пример решения

Метод Жордана Гаусса

Найдем алгебраические дополнения ко всем элементам матрицы А:

Пример 2.

Запишем матрицу

Найдем матрицу

Сделаем проверку:

Пример 3.

Найти матрицу, обратную к матрице А

1) Найдем Матрица существует, только если

2) Найдем алгебраические дополнения к элементам матрицы А:

3) Запишем присоединенную матрицу:

Итак, для матрицы 2-го порядка присоединенная матрица находится очень просто — элементы главной диагонали меняются местами, а элементы побочной диагонали умножаются на (-1):

4) Найдем обратную матрицу 7.1. Типичные задачи Матричные уравнения естественно возникают в задачах, которые изначально выглядят, как «векторные».

Скажем, поиск собственных векторов объединяется одним уравнением (7.1) где — искомая матрица со столбцами в качестве собственных векторов. Другой характерный пример — линейное дифференциальное уравнение

Как известно, общее решение имеет вид где линейно независимые решения как вектор-столбцы, составляют матрицу фундаментальных решений удовлетворяющую матричному дифференциальному уравнению которое выгоднее рассматривать с самого начала вместо

Поиск преобразования X, обеспечивающего подобие матриц, порождает уравнение После умножения слева на X оно переходит в эквивалентное (7.

2) в предположении невырожденности X. Очевидно, (7.1) при заданной матрице Л представляет собой частный случай (7.2). Наконец, поиск функции Ляпуновадля линейной системы приводит к уравнению (7.3) относительно матрицы V.

Обозначая неизвестную матрицу V через X и обобщая (7.3), приходим к уравнению (7.4) которое охватывает в качестве частных случаев все рассмотренные выше случаи, — разумеется, кроме дифференциального. Уравнение (7.4) линейно относительно элементов неизвестной матрицы X, и этим замечанием, казалось бы, можно закончить исследование, сославшись на предыдущее изучение линейных уравнений.

Но проблема заключается в том, что уравнение (7.4), как линейное, имеет нестандартную форму, опираясь на двухин-дексное описание переменных.

В принципе, нет никакой трудности в том, чтобы перенумеровать переменные вытянув их в строчку. Но при бесхитростной перенумерации содержательная информация о матрицах A, J9, С может разрушиться, что будет означать отсутствие смысловой связи между получаемыми линейными системами и исходными операторами действующими в Для решения таких задач имеется специальный инструмент — кронекерово произведение2) матриц, Если А и В — прямоугольные матрицы размера, соответственно, то

Размер

Свойства легко проверяются.

Важную роль играет формула (7.5) Легко проверяется Менее очевидно, что в случае невырожденности квадратных матриц А и В произведение тоже невырожденно) Если теперь допустить кратные собственные значения у А и В, то идея предельного перехода здесь работает без проблем. Собственные векторы в пределе могут становиться линейно зависимыми, но это в данном случае ничему не мешает.

Поэтому утверждение 7.2.1 справедливо без каких бы то ни было предположений о матрицах А и В. Сразу становится ясной отмечавшаяся выше невырожденность в случае невырожденности А и В.

Обычное соотношение может быть записано в виде где вектор это вытянутая в столбик матрица вектор из получен аналогично. Соотношение записывается иначе, Поэтому уравнение (7.4), с помощью кронекерова произведения можно переписать так { (7.7)

В этой перезаписи уравнения не было бы большого смысла, если бы она не позволяла делать выводы в терминах исходных матриц А и В.

Но специфика кронекерова произведения как раз такова, что она дает возможность судить о спектральных свойствах «®-матриц» во многих практических ситуациях. Причиной является следующий факт. Лемма. Если тo Результат сразу вытекает из (7.6),

Лемма 7.3.1 означает, что матрицы А и В в могут быть приведены к желаемому виду (диагональному, треугольному, жордановому) независимо друг от друга.

Пусть, например. где — соответствующие жордановы формы. Тогда ясно, что спектры и что еше раз доказывает утверждение 7.2.1. Точно так же А и В могут быть приведены к своим жордановым формам) в (7.8) Отсюда ясно, что для невырожденности (7.8) необходимо и достаточно, чтобы не нашлось противоположных собственных значений, Это и является условием однозначной разрешимости уравнения (7.7), т.е. (7.4).

Как теоретический инструмент иногда полезна формула (7.9) дающая решение уравнения в случае, когда матрицы А и В гурвицевы, т.е. действительные части их собственных значений строго отрицательны. Устанавливается это совсем легко. Решением задачи Коши (7.10) является что проверяется подстановкой.

Из гурвицевости А и В следует экспоненциально быстрое убывание до нуля при Это позволяет проинтегрировать (7.

10) от 0 до оо, что сразу дает (7.9). В частности, решение уравнения в случае гурвицевой матрицы А приводит к положительно определенной функции Ляпунова Найти (методом элементарных преобразований) матрицу, обратную к данной:

Записывая матрицу размера (3 х 6), с помощью элементарных преобразований над строками приведем ее сначала к ступенчатому виду а затем к виду

Итак,

Сделаем проверку:

21. Матричные уравнения. Теорема существования и единственности решения.

Рассмотрим матричное уравнение вида

где и — данные матрицы, имеющие одинаковое количество строк, причем матрица квадратная. Требуется найти матрицу , удовлетворяющую уравнению (4.5).

Теорема 4.2 о существовании и единственности решения матричного уравнения (4.5). Если определитель матрицы отличен от нуля, то матричное уравнение (4.5) имеет единственное решение.

В самом деле, подставляя в левую часть равенства (4. 5), получаем, т.е. правую часть этого равенства.

Заметим, что решением матричного уравнения служит обратная матрица.

Рассмотрим также матричное уравнение вида

где и — данные матрицы, имеющие одинаковое количество столбцов, причем матрица квадратная. Требуется найти матрицу , удовлетворяющую уравнению (4.6).

Теорема 4.3 о существовании и единственности решения матричного уравнения (4.6). Если определитель матрицы отличен от нуля, то уравнение (4.6) имеет единственное решение.

Заметим, что матрица является как бы «левым» частным от «деления» матрицына матрицу, поскольку матрицав (4.5) умножается наслева, а матрица— «правым» частным, так как матрицав (4.6) умножается насправа.

Пример 4.5. Даны матрицы

Решить уравнения: а) ; б); в).

Решение. Обратная матрица была найдена в примере 4.2.

а) Решение уравнения находим, умножая обе его части слева на

б) Уравнение не имеет решений, так как матрицы иимеют разное количество столбцов.

в) Решение уравнения находим, умножая обе его части справа на

Пример 4.6. Решить уравнение: , где.

Решение. Преобразуя левую часть уравнения:

приведем его к виду (4.1)

 где 

Следовательно, . Обратная матрица найдена в примере 4.2:

 Значит, 

Пример 4.7. Решить уравнение , где

Решение. Обратные матрицы были найдены в примерах 4.2, 4.3 соответственно. Решение уравнения находим по формуле

Пример 4.8. Решить уравнение , где

Решение. Определитель матрицы равен нулю, следовательно, обратная матрица не существует. Поэтому нельзя использовать формулу. Будем искать элементы матрицы. Подставляя в уравнение, получаем

Находим произведение, а затем приравниваем соответствующие элементы матриц в левой и правой частях уравнения:

Здесь, учитывая пропорциональность уравнений, в системе оставлены только два уравнения из четырех. Выразим неизвестные и

Следовательно, решение матричного уравнения имеет вид

где параметры и могут принимать любые значения. Таким образом, данное матричное уравнение имеет бесконечное множество решений.

Рассмотрим систему уравнений

— матрица системы

— матрицы-столбцы неизвестных и свободных членов.

Очевидно, что ,

тогда АХ=С

Такое равенство называется матричным уравнением.

Если матрица А системы невырожденная, (det А 0), то это уравнение решается следующим образом:

Умножим обе его части на матрицу А-1, обратную матрице А

А-1(АХ)=А-1С или,

-1А) · Х = А-1·С. но так как А-1А=Е, и ЕХ=Х Х=А-1С

Например, решим матричным способом систему

матрица системы

Не является ли матрица А вырожденной? Найдем ее определитель:

 А =1·[-1·4 – 1·2] – 1·[2·4 – 2·4] + 2·[2·1 – 4·(-1)] = -6 + 12 = 6

Определитель не равен нулю, то есть матрица не вырожденная. Значит, существует обратная матрица

А11 = (-1)1+1·М11 = (+1)·[-1·4 – 1·2] = -6

А12 = (-1)1+2·М12 = (-1)·[2·4 – 2·4] = 0

А13 = (-1)1+3·М13 = (+1)·[2·1 – 4·(-1)] = 6

А21 = (-1)2+1·М21 = (-1)·[1·4 – 1·2] = -2

А22 = (-1)2+2·М22 = [1·4 – 2·4] = -4

А23 = (-1)2+3·М23 = (-1)·[1·1 – 4·1] = 3

А31 = (-1)3+1М31 = [1·2 – (-1)·2] = 4

А32 = (-1)3+2·М32 = [(-1)·1·2 – 2·2] = 2

А33 = (-1)3+3·М33 = [1·(-1) – 2·1] = -3

Можно убедиться проверкой в правильности решения: подставим вектор Х в первоначальное матричное уравнение.

Действительно вектор Х удовлетворяет заданной системе

   Решение систем уравнений методом Крамера

Применим теперь наши знания о матрицах к решению систем уравнений первой степени. Рассмотрим систему двух уравнений с двумя неизвестными:

или коротко или АХ=С

система записана в матричном виде (как произведение матриц)

Решим эту простенькую систему школьными методами.

Умножим первое уравнение на а22, а второе на (-а12) и сложим

11а22 – а21а121 = с1а22 – с2а12

аналогично

11а22 – а21а122 = с2а11 – с1а21

1) но а11а22 – а21а12 = — это определитель матрицы А(det А) или его еще называют определитель системы и он составлен из коэффициентов при неизвестных. Обозначим его 

2) 

определитель, который получится из det А, если в нем столбец коэффициентов при х1 (первый столбец) заменить на столбец правых частей. Обозначим его  Х1

3) 

  • определитель, который получится, если в det А столбец

  • коэффициентов при х2 заменить на столбец правых частей. Обозначим его  x2

Видим, что <=»» font=»»>

Как вы понимаете, если мы возьмем систему трех уравнений с тремя неизвестными или n уравнений с n неизвестными, то формулы останутся те же:

Эти формулы широко известны и называются формулами Крамера. Мы же с Вами займемся анализом того существует ли решение и единственно ли оно?

Возможны 3 случая:

1.  0 Тогда xi= xi/ — решение существует, причем единственное.

2.  =0 , а какой-либо из  xi 0 , то есть у нас в xi= xi/ производится деление на 0, система не имеет решения (несовместна).

3.  =0 и все  xi=0 то система  имеет бесконечно много решений.

Пример:

Так как второе уравнение получается из первого умножением на 2, то наша система равносильна такой системе.

Так получилось, потому что первое и второе уравнения систем эквивалентны и фактически мы имеем систему двух уравнений с тремя неизвестными, то есть неопределенную систему. Она имеет бесчисленное множество решений. Положив, например, z=0

получим систему

Решив ее, найдем 11х=0, х=0, y=1

То есть решение первоначальной системы x=0, y=0, z=0.

Если бы мы положили z=1, получили бы еще один ответ и так далее.

Теорема (правило Крамера). Если определитель системы Δ ≠ 0, то рассматриваемая система имеет одно и только одно решение, причём

Доказательство. Итак, рассмотрим систему 3-х уравнений с тремя неизвестными. Умножим 1-ое уравнение системы на алгебраическое дополнение A11 элемента a11, 2-ое уравнение – наA21 и 3-е – на A31:

Сложим эти уравнения:

Рассмотрим каждую из скобок и правую часть этого уравнения. По теореме о разложении определителя по элементам 1-го столбца

.

Далее рассмотрим коэффициенты при x2:

Аналогично можно показать, что и .

Наконец несложно заметить, что 

Таким образом, получаем равенство: .

Следовательно, .

Аналогично выводятся равенства и , откуда и следует утверждение теоремы.

Таким образом, заметим, что если определитель системы Δ ≠ 0, то система имеет единственное решение и обратно. Если же определитель системы равен нулю, то система либо имеет бесконечное множество решений, либо не имеет решений, т.е. несовместна.

Axb c матрицы уравнения — ПК портал

Содержание

  1. Решение матричных уравнений: как это делается
  2. Решение матричных уравнений: примеры
  3. AX = B, где матрица A обратима
  4. XA = B, где матрица A обратима

Решение матричных уравнений: как это делается

Матричные уравнения имеют прямую аналогию с простыми алгебраическими уравнениями, в которых присутствует операция умножения. Например,

где x — неизвестное.

А, поскольку мы уже умеем находить произведение матриц, то можем приступать к рассмотрению аналогичных уравнений с матрицами, в которых буквы — это матрицы.

Итак, матричным уравнением называется уравнение вида

где A и B — известные матрицы, X — неизвестная матрица, которую требуется найти.

Как решить матричное уравнение в первом случае? Для того, чтобы решить матричное уравнение вида AX = B , обе его части следует умножить на обратную к A матрицу слева:

.

По определению обратной матрицы, произведение обратной матрицы на данную исходную матрицу равно единичной матрице: , поэтому

.

Так как E — единичная матрица, то EX = X . В результате получим, что неизвестная матрица X равна произведению матрицы, обратной к матрице A , слева, на матрицу B :

.

Как решить матричное уравнение во втором случае? Если дано уравнение

то есть такое, в котором в произведении неизвестной матрицы X и известной матрицы A матрица A находится справа, то нужно действовать аналогично, но меняя направление умножения на матрицу, обратную матрице A , и умножать матрицу B на неё справа:

,

,

.

Как видим, очень важно, с какой стороны умножать на обратную матрицу, так как . Обратная к A матрица умножается на матрицу B с той стороны, с которой матрица A умножается на неизвестную матрицу X . То есть с той стороны, где в произведении с неизвестной матрицей находится матрица A .

Как решить матричное уравнение в третьем случае? Встречаются случаи, когда в левой части уравнения неизвестная матрица X находится в середине произведения трёх матриц. Тогда известную матрицу из правой части уравнения следует умножить слева на матрицу, обратную той, которая в упомянутом выше произведении трёх матриц была слева, и справа на матрицу, обратную той матрице, которая располагалась справа. Таким образом, решением матричного уравнения

.

Решение матричных уравнений: примеры

Пример 1. Решить матричное уравнение

.

Решение. Данное уравнение имеет вид AX = B , то есть в произведении матрицы A и неизвестной матрицы X матрица A находится слева. Поэтому решение следует искать в виде , то есть неизвестная матрица равна произведению матрицы B на матрицу, обратную матрице A слева. Найдём матрицу, обратную матрице A .

Сначала найдём определитель матрицы A :

.

Найдём алгебраические дополнения матрицы A :

.

Составим матрицу алгебраических дополнений:

.

Транспонируя матрицу алгебраических дополнений, находим матрицу, союзную с матрицей A :

.

Теперь у нас есть всё, чтобы найти матрицу, обратную матрице A :

.

Наконец, находим неизвестную матрицу:

Пример 2. Решить матричное уравнение

.

Пример 3. Решить матричное уравнение

.

Решение. Данное уравнение имеет вид XA = B , то есть в произведении матрицы A и неизвестной матрицы X матрица A находится справа. Поэтому решение следует искать в виде , то есть неизвестная матрица равна произведению матрицы B на матрицу, обратную матрице A справа. Найдём матрицу, обратную матрице A .

Сначала найдём определитель матрицы A :

.

Найдём алгебраические дополнения матрицы A :

.

Составим матрицу алгебраических дополнений:

.

Транспонируя матрицу алгебраических дополнений, находим матрицу, союзную с матрицей A :

.

Находим матрицу, обратную матрице A :

.

Находим неизвестную матрицу:

До сих пор мы решали уравнения с матрицами второго порядка, а теперь настала очередь матриц третьего порядка.

Пример 4. Решить матричное уравнение

.

Решение. Это уравнение первого вида: AX = B , то есть в произведении матрицы A и неизвестной матрицы X матрица A находится слева. Поэтому решение следует искать в виде , то есть неизвестная матрица равна произведению матрицы B на матрицу, обратную матрице A слева. Найдём матрицу, обратную матрице A .

Сначала найдём определитель матрицы A :

.

Найдём алгебраические дополнения матрицы A :

Составим матрицу алгебраических дополнений:

Транспонируя матрицу алгебраических дополнений, находим матрицу, союзную с матрицей A :

.

Находим матрицу, обратную матрице A , и делаем это легко, так как определитель матрицы A равен единице:

.

Находим неизвестную матрицу:

Пример 5. Решить матричное уравнение

.

Решение. Данное уравнение имеет вид XA = B , то есть в произведении матрицы A и неизвестной матрицы X матрица A находится справа. Поэтому решение следует искать в виде , то есть неизвестная матрица равна произведению матрицы B на матрицу, обратную матрице A справа. Найдём матрицу, обратную матрице A .

Сначала найдём определитель матрицы A :

.

Найдём алгебраические дополнения матрицы A :

Составим матрицу алгебраических дополнений:

.

Транспонируя матрицу алгебраических дополнений, находим матрицу, союзную с матрицей A :

.

Находим матрицу, обратную матрице A :

.

Находим неизвестную матрицу:

Пример 6. Решить матричное уравнение

.

Решение. Данное уравнение имеет вид AXB = C , то есть неизвестная матрица X находится в середине произведения трёх матриц. Поэтому решение следует искать в виде . Найдём матрицу, обратную матрице A .

Сначала найдём определитель матрицы A :

.

Найдём алгебраические дополнения матрицы A :

.

Составим матрицу алгебраических дополнений:

.

Транспонируя матрицу алгебраических дополнений, находим матрицу, союзную с матрицей A :

.

Находим матрицу, обратную матрице A :

.

Найдём матрицу, обратную матрице B .

Сначала найдём определитель матрицы B :

.

Найдём алгебраические дополнения матрицы B :

Составим матрицу алгебраических дополнений матрицы B :

.

Транспонируя матрицу алгебраических дополнений, находим матрицу, союзную с матрицей B :

.

Находим матрицу, обратную матрице B :

.

Рассмотрим матричное уравнение вида

где [math]A[/math] и [math]B[/math] — данные матрицы, имеющие одинаковое количество строк, причем матрица [math]A[/math] квадратная. <-1>= egin-5 & 3\ 2 & -1 end ightarrow X= egin3 & 5\ 2 & 1 endcdot egin-5 & 3\ 2 & -1 end= egin-5 & 4\ -8 & 5 end$

Решение алгебраических линейных уравнений матричным методом. Решение систем линейных алгебраических уравнений с помощью обратной матрицы

Метод обратной матрицы – эточастный случай матричного уравнения

Решить систему с матричным методом

Решение : Запишем систему в матричной форме.Решение системы найдем по формуле (см.последнюю формулу)

Обратную матрицу найдем по формуле:
, где – транспонированная матрица алгебраических дополнений соответствующих элементов матрицы .

Сначала разбираемся с определителем:

Здесь определитель раскрыт по первой строке.

Внимание! Если, то обратной матрицы не существует, и решить систему матричным методом невозможно. В этом случае система решаетсяметодом исключение неизвестных (методом Гаусса).

Теперь нужно вычислить 9 миноров и записать их в матрицу миноров

Справка: Полезно знать смысл двойных подстрочных индексов в линейной алгебре. Первая цифра – это номер строки, в которой находится данный элемент. Вторая цифра – это номер столбца, в котором находится данный элемент:

То есть, двойной подстрочный индекс указывает, что элемент находится в первой строке, третьем столбце, а, например, элемент находится в 3 строке, 2 столбце

В ходе решения расчет миноров лучше расписать подробно, хотя, при определенном опыте их можно приноровиться считать с ошибками устно.






Порядок расчета миноров совершенно не важен, здесь я их вычислил слева направо по строкам. Можно было рассчитать миноры по столбцам (это даже удобнее).

Таким образом:

– матрица миноров соответствующих элементов матрицы .

– матрица алгебраических дополнений.

– транспонированная матрица алгебраических дополнений.

Повторюсь, выполненные шаги мы подробно разбирали на уроке Как найти обратную матрицу?

Теперь записываем обратную матрицу:

Ни в коем случае не вносимв матрицу, это серьезно затруднит дальнейшие вычисления . Деление нужно было бы выполнить, если бы все числа матрицы делились на 60 без остатка. А вот внести минус в матрицу в данном случае очень даже нужно, это, наоборот – упростит дальнейшие вычисления.

Осталось провести матричное умножение. Умножать матрицы можно научиться на урокеДействия с матрицами . Кстати, там разобран точно такой же пример.

Обратите внимание, что деление на 60 выполняется в последнюю очередь .
Иногда может и не разделиться нацело, т.е. могут получиться «плохие» дроби. Что в таких случаях делать, я уже рассказал, когда мы разбирали правило Крамера.

Ответ :

Пример 12

Решить систему с помощью обратной матрицы.

Это пример для самостоятельного решения (образец чистового оформления и ответ в конце урока).

Наиболее универсальным способом решения системы является метод исключения неизвестных (метод Гаусса) . Доступно объяснить алгоритм не так-то просто, но я старался!.

Желаю успехов!

Ответы:

Пример 3:

Пример 6:

Пример 8: , . Вы можете посмотреть или скачать образец решения данного примера (ссылка ниже).

Примеры 10, 12:

Продолжаем рассматривать системы линейных уравнений. Этот урок является третьим по теме. Если Вы смутно представляете, что такое система линейных уравнений вообще, чувствуете себя чайником, то рекомендую начать с азов на странице Далее полезно изучить урок .

Метод Гаусса – это просто! Почему? Известный немецкий математик Иоганн Карл Фридрих Гаусс еще при жизни получил признание величайшего математика всех времен, гения и даже прозвище «короля математики». А всё гениальное, как известно – просто! Кстати, на деньги попадают не только лохи, но еще и гении – портрет Гаусса красовался на купюре в 10 дойчмарок (до введения евро), и до сих пор Гаусс загадочно улыбается немцам с обычных почтовых марок.

Метод Гаусса прост тем, что для его освоения ДОСТАТОЧНО ЗНАНИЙ ПЯТИКЛАССНИКА.Необходимо уметь складывать и умножать! Не случайно метод последовательного исключения неизвестных преподаватели часто рассматривают на школьных математических факультативах. Парадокс, но у студентов метод Гаусса вызывает наибольшие сложности. Ничего удивительного – всё дело в методике, и я постараюсь в доступной форме рассказать об алгоритме метода.

Сначала немного систематизируем знания о системах линейных уравнений. Система линейных уравнений может:

1) Иметь единственное решение.
2) Иметь бесконечно много решений.
3) Не иметь решений (быть несовместной ).

Метод Гаусса – наиболее мощный и универсальный инструмент для нахождения решениялюбой системы линейных уравнений. Как мы помним, правило Крамера и матричный метод непригодны в тех случаях, когда система имеет бесконечно много решений или несовместна. А метод последовательного исключения неизвестных в любом случае приведет нас к ответу! На данном уроке мы опять рассмотрим метод Гаусса для случая №1 (единственное решение системы), под ситуации пунктов №№2-3 отведена статья. Замечу, что сам алгоритм метода во всех трёх случаях работает одинаково.

Вернемся к простейшей системе с урока Как решить систему линейных уравнений?
и решим ее методом Гаусса.

На первом этапе нужно записать расширенную матрицу системы :
. По какому принципу записаны коэффициенты, думаю, всем видно. Вертикальная черта внутри матрицы не несёт никакого математического смысла – это просто отчеркивание для удобства оформления.

Справка: рекомендую запомнить термины линейной алгебры. Матрица системы – это матрица, составленная только из коэффициентов при неизвестных, в данном примере матрица системы: . Расширенная матрица системы – это та же матрица системы плюс столбец свободных членов, в данном случае: . Любую из матриц можно для краткости называть просто матрицей.

После того, как расширенная матрица система записана, с ней необходимо выполнить некоторые действия, которые также называются элементарными преобразованиями .

Существуют следующие элементарные преобразования:

1) Строки матрицы можно переставлять местами. Например, в рассматриваемой матрице можно безболезненно переставить первую и вторую строки:

2) Если в матрице есть (или появились) пропорциональные (как частный случай – одинаковые) строки, то следует удалить из матрицы все эти строки кроме одной. Рассмотрим, например матрицу . В данной матрице последние три строки пропорциональны, поэтому достаточно оставить только одну из них: .

3) Если в матрице в ходе преобразований появилась нулевая строка, то ее также следуетудалить . Рисовать не буду, понятно, нулевая строка – это строка, в которой одни нули .

4) Строку матрицы можно умножить (разделить) на любое число, отличное от нуля . Рассмотрим, например, матрицу . Здесь целесообразно первую строку разделить на –3, а вторую строку – умножить на 2: . Данное действие очень полезно, поскольку упрощает дальнейшие преобразования матрицы.

5) Это преобразование вызывает наибольшие затруднения, но на самом деле ничего сложного тоже нет. К строке матрицы можно прибавить другую строку, умноженную на число , отличное от нуля. Рассмотрим нашу матрицу из практического примера: . Сначала я распишу преобразование очень подробно. Умножаем первую строку на –2: , и ко второй строке прибавляем первую строку умноженную на –2 : . Теперь первую строку можно разделить «обратно» на –2: . Как видите, строка, которую ПРИБАВЛЯЛИ не изменилась . Всегда меняется строка, К КОТОРОЙ ПРИБАВЛЯЮТ .

На практике так подробно, конечно, не расписывают, а пишут короче:

Еще раз: ко второй строке прибавили первую строку, умноженную на –2 . Умножают строку обычно устно или на черновике, при этом мысленный ход расчётов примерно такой:

«Переписываю матрицу и переписываю первую строку: »

«Сначала первый столбец. Внизу мне нужно получить ноль. Поэтому единицу вверху умножаю на –2: , и ко второй строке прибавляю первую: 2 + (–2) = 0. Записываю результат во вторую строку: »

«Теперь второй столбец. Вверху –1 умножаю на –2: . Ко второй строке прибавляю первую: 1 + 2 = 3. Записываю результат во вторую строку: »

«И третий столбец. Вверху –5 умножаю на –2: . Ко второй строке прибавляю первую: –7 + 10 = 3. Записываю результат во вторую строку: »

Пожалуйста, тщательно осмыслите этот пример и разберитесь в последовательном алгоритме вычислений, если вы это поняли, то метод Гаусса практически «в кармане». Но, конечно, над этим преобразованием мы еще поработаем.

Элементарные преобразования не меняют решение системы уравнений

! ВНИМАНИЕ: рассмотренные манипуляции нельзя использовать , если Вам предложено задание, где матрицы даны «сами по себе». Например, при «классических» действиях с матрицами что-то переставлять внутри матриц ни в коем случае нельзя!

Вернемся к нашей системе . Она уже почти решена.

Запишем расширенную матрицу системы и с помощью элементарных преобразований приведем ее к ступенчатому виду :

(1) Ко второй строке прибавили первую строку, умноженную на –2. Кстати, почему первую строку умножаем именно на –2? Для того чтобы внизу получить ноль, а значит, избавиться от одной переменной во второй строке.

(2) Делим вторую строку на 3.

Цель элементарных преобразований привести матрицу к ступенчатому виду: . В оформлении задания прямо так и отчеркивают простым карандашом «лестницу», а также обводят кружочками числа, которые располагаются на «ступеньках». Сам термин «ступенчатый вид» не вполне теоретический, в научной и учебной литературе он часто называется трапециевидный вид или треугольный вид .

В результате элементарных преобразований получена эквивалентная исходной система уравнений:

Теперь систему нужно «раскрутить» в обратном направлении – снизу вверх, этот процесс называется обратным ходом метода Гаусса .

В нижнем уравнении у нас уже готовый результат: .

Рассмотрим первое уравнение системы и подставим в него уже известное значение «игрек»:

Рассмотрим наиболее распространенную ситуацию, когда методом Гаусса требуется решить систему трёх линейных уравнений с тремя неизвестными.

Пример 1

Решить методом Гаусса систему уравнений:

Запишем расширенную матрицу системы:

Сейчас я сразу нарисую результат, к которому мы придём в ходе решения:

И повторюсь, наша цель – с помощью элементарных преобразований привести матрицу к ступенчатому виду. С чего начать действия?

Сначала смотрим на левое верхнее число:

Почти всегда здесь должна находиться единица . Вообще говоря, устроит и –1 (а иногда и другие числа), но как-то так традиционно сложилось, что туда обычно помещают единицу. Как организовать единицу? Смотрим на первый столбец – готовая единица у нас есть! Преобразование первое: меняем местами первую и третью строки:

Теперь первая строка у нас останется неизменной до конца решения . Уже легче.

Единица в левом верхнем углу организована. Теперь нужно получить нули вот на этих местах:

Нули получаем как раз с помощью «трудного» преобразования. Сначала разбираемся со второй строкой (2, –1, 3, 13). Что нужно сделать, чтобы на первой позиции получить ноль? Нужно ко второй строке прибавить первую строку, умноженную на –2 . Мысленно или на черновике умножаем первую строку на –2: (–2, –4, 2, –18). И последовательно проводим (опять же мысленно или на черновике) сложение, ко второй строке прибавляем первую строку, уже умноженную на –2 :

Результат записываем во вторую строку:

Аналогично разбираемся с третьей строкой (3, 2, –5, –1). Чтобы получить на первой позиции ноль, нужно к третьей строке прибавить первую строку, умноженную на –3 . Мысленно или на черновике умножаем первую строку на –3: (–3, –6, 3, –27). И к третьей строке прибавляем первую строку, умноженную на –3 :

Результат записываем в третью строку:

На практике эти действия обычно выполняются устно и записываются в один шаг:

Не нужно считать всё сразу и одновременно . Порядок вычислений и «вписывания» результатов последователен и обычно такой: сначала переписываем первую строку, и пыхтим себе потихонечку – ПОСЛЕДОВАТЕЛЬНО иВНИМАТЕЛЬНО :

А мысленный ход самих расчётов я уже рассмотрел выше.

В данном примере это сделать легко, вторую строку делим на –5 (поскольку там все числа делятся на 5 без остатка). Заодно делим третью строку на –2, ведь чем меньше числа, тем проще решение:

На заключительном этапе элементарных преобразований нужно получить еще один ноль здесь:

Для этого к третьей строке прибавляем вторую строку, умноженную на –2 :

Попробуйте разобрать это действие самостоятельно – мысленно умножьте вторую строку на –2 и проведите сложение.

Последнее выполненное действие – причёска результата, делим третью строку на 3.

В результате элементарных преобразований получена эквивалентная исходной система линейных уравнений:

Круто.

Теперь в действие вступает обратный ход метода Гаусса. Уравнения «раскручиваются» снизу вверх.

В третьем уравнении у нас уже готовый результат:

Смотрим на второе уравнение: . Значение «зет» уже известно, таким образом:

И, наконец, первое уравнение: . «Игрек» и «зет» известны, дело за малым:

Ответ:

Как уже неоднократно отмечалось, для любой системы уравнений можно и нужно сделать проверку найденного решения, благо, это несложно и быстро.

Пример 2


Это пример для самостоятельного решения, образец чистового оформления и ответ в конце урока.

Следует отметить, что ваш ход решения может не совпасть с моим ходом решения, и это – особенность метода Гаусса . Но вот ответы обязательно должны получиться одинаковыми!

Пример 3

Решить систему линейных уравнений методом Гаусса

Запишем расширенную матрицу системы и с помощью элементарных преобразований приведем ее к ступенчатому виду:

Смотрим на левую верхнюю «ступеньку». Там у нас должна быть единица. Проблема состоит в том, что в первом столбце единиц нет вообще, поэтому перестановкой строк ничего не решить. В таких случаях единицу нужно организовать с помощью элементарного преобразования. Обычно это можно сделать несколькими способами. Я поступил так: (1) К первой строке прибавляем вторую строку, умноженную на –1 . То есть, мысленно умножили вторую строку на –1 и выполнили сложение первой и второй строки, при этом вторая строка у нас не изменилась.

Теперь слева вверху –1, что нас вполне устроит. Кто хочет получить +1, может выполнить дополнительное телодвижение: умножить первую строку на –1 (сменить у неё знак).

(2) Ко второй строке прибавили первую строку, умноженную на 5. К третьей строке прибавили первую строку, умноженную на 3.

(3) Первую строку умножили на –1, в принципе, это для красоты. У третьей строки также сменили знак и переставили её на второе место, таким образом, на второй «ступеньке у нас появилась нужная единица.

(4) К третьей строке прибавили вторую строку, умноженную на 2.

(5) Третью строку разделили на 3.

Скверным признаком, который свидетельствует об ошибке в вычислениях (реже – об опечатке), является «плохая» нижняя строка. То есть, если бы у нас внизу получилось что-нибудь вроде , и, соответственно, , то с большой долей вероятности можно утверждать, что допущена ошибка в ходе элементарных преобразований.

Заряжаем обратный ход, в оформлении примеров часто не переписывают саму систему, а уравнения «берут прямо из приведенной матрицы». Обратный ход, напоминаю, работает, снизу вверх:
Да тут подарок получился:

Ответ: .

Пример 4

Решить систему линейных уравнений методом Гаусса

Это пример для самостоятельного решения, он несколько сложнее. Ничего страшного, если кто-нибудь запутается. Полное решение и образец оформления в конце урока. Ваше решение может отличаться от моего решения.

В последней части рассмотрим некоторые особенности алгоритма Гаусса.
Первая особенность состоит в том, что иногда в уравнениях системы отсутствуют некоторые переменные, например:

Как правильно записать расширенную матрицу системы? Об этом моменте я уже рассказывал на уроке Правило Крамера. Матричный метод . В расширенной матрице системы на месте отсутствующих переменных ставим нули:

Кстати, это довольно легкий пример, поскольку в первом столбце уже есть один ноль, и предстоит выполнить меньше элементарных преобразований.

Вторая особенность состоит вот в чём. Во всех рассмотренных примерах на «ступеньки» мы помещали либо –1, либо +1. Могут ли там быть другие числа? В ряде случаев могут. Рассмотрим систему: .

Здесь на левой верхней «ступеньке» у нас двойка. Но замечаем тот факт, что все числа в первом столбце делятся на 2 без остатка – и другая двойка и шестерка. И двойка слева вверху нас устроит! На первом шаге нужно выполнить следующие преобразования: ко второй строке прибавить первую строку, умноженную на –1; к третьей строке прибавить первую строку, умноженную на –3. Таким образом, мы получим нужные нули в первом столбце.

Или еще такой условный пример: . Здесь тройка на второй «ступеньке» тоже нас устраивает, поскольку 12 (место, где нам нужно получить ноль) делится на 3 без остатка. Необходимо провести следующее преобразование: к третьей строке прибавить вторую строку, умноженную на –4, в результате чего и будет получен нужный нам ноль.

Метод Гаусса универсален, но есть одно своеобразие. Уверенно научиться решать системы другими методами (методом Крамера, матричным методом) можно буквально с первого раза – там очень жесткий алгоритм. Но вот чтобы уверенно себя чувствовать в методе Гаусса, следует «набить руку», и прорешать хотя бы 5-10 десять систем. Поэтому поначалу возможны путаница, ошибки в вычислениях, и в этом нет ничего необычного или трагического.

Дождливая осенняя погода за окном…. Поэтому для всех желающих более сложный пример для самостоятельного решения:

Пример 5

Решить методом Гаусса систему 4-х линейных уравнений с четырьмя неизвестными.

Такое задание на практике встречается не так уж и редко. Думаю, даже чайнику, который обстоятельно изучил эту страницу, интуитивно понятен алгоритм решения такой системы. Принципиально всё так же – просто действий больше.

Случаи, когда система не имеет решений (несовместна) или имеет бесконечно много решений, рассмотрены на уроке Несовместные системы и системы с общим решением . Там же можно закрепить рассмотренный алгоритм метода Гаусса.

Желаю успехов!

Решения и ответы:

Пример 2: Запишем расширенную матрицу системы и с помощью элементарных преобразований приведем ее к ступенчатому виду.

Выполненные элементарные преобразования:
(1) Ко второй строке прибавили первую строку, умноженную на –2. К третьей строке прибавили первую строку, умноженную на –1. Внимание! Здесь может возникнуть соблазн из третьей строки вычесть первую, крайне не рекомендую вычитать – сильно повышается риск ошибки. Только складываем!
(2) У второй строки сменили знак (умножили на –1). Вторую и третью строки поменяли местами. Обратите внимание , что на «ступеньках» нас устраивает не только единица, но еще и –1, что даже удобнее.
(3) К третьей строке прибавили вторую строку, умноженную на 5.
(4) У второй строки сменили знак (умножили на –1). Третью строку разделили на 14.

Обратный ход:


Ответ: .

Пример 4: Запишем расширенную матрицу системы и с помощью элементарных преобразований приведем ее к ступенчатому виду:

Выполненные преобразования:
(1) К первой строке прибавили вторую. Таким образом, организована нужная единица на левой верхней «ступеньке».
(2) Ко второй строке прибавили первую строку, умноженную на 7. К третьей строке прибавили первую строку, умноженную на 6.

Со второй «ступенькой» всё хуже , «кандидаты» на неё – числа 17 и 23, а нам нужна либо единичка, либо –1. Преобразования (3) и (4) будут направлены на получение нужной единицы

(3) К третьей строке прибавили вторую, умноженную на –1.
(4) Ко второй строке прибавили третью, умноженную на –3.
Нужная вещь на второй ступеньке получена .
(5) К третьей строке прибавили вторую, умноженную на 6.
(6) Вторую строку умножили на –1, третью строку разделили на -83. .Очевидно, что плоскость однозначно определяется тремя различными точками, не лежащими на одной прямой. Поэтому достаточно популярны трёхбуквенные обозначения плоскостей – по принадлежащим им точкам, например, ; .Если свободные члены

Пусть имеется квадратная матрица n-го порядка

Матрица А -1 называется обратной матрицей по отношению к матрице А, если А*А -1 = Е, где Е — единичная матрица n-го порядка.

Единичная матрица — такая квадратная матрица, у которой все элементы по главной диагонали, проходящей от левого верхнего угла к правому нижнему углу, — единицы, а остальные — нули, например:

Обратная матрица может существовать только для квадратных матриц т.е. для тех матриц, у которых число строк и столбцов совпадают.

Теорема условия существования обратной матрицы

Для того чтобы матрица имела обратную матрицу необходимо и достаточно, чтобы она была невырожденной.

Матрица А = (А1, А2,…А n) называется невырожденной , если векторы-столбцы являются линейно независимыми. Число линейно независимых векторов-столбцов матрицы называется рангом матрицы . Поэтому можно сказать, что для того, чтобы существовала обратная матрица, необходимо и достаточно, чтобы ранг матрицы равнялся ее размерности, т.е. r = n.

Алгоритм нахождения обратной матрицы

  1. Записать в таблицу для решения систем уравнений методом Гаусса матрицу А и справа (на место правых частей уравнений) приписать к ней матрицу Е.
  2. Используя преобразования Жордана, привести матрицу А к матрице, состоящей из единичных столбцов; при этом необходимо одновременно преобразовать матрицу Е.
  3. Если необходимо, то переставить строки (уравнения) последней таблицы так, чтобы под матрицей А исходной таблицы получилась единичная матрица Е.
  4. Записать обратную матрицу А -1 , которая находится в последней таблице под матрицей Е исходной таблицы.

Пример 1

Для матрицы А найти обратную матрицу А -1

Решение: Записываем матрицу А и справа приписываем единичную матрицу Е. Используя преобразования Жордана, приводим матрицу А к единичной матрице Е. Вычисления приведены в таблице 31.1.

Проверим правильность вычислений умножением исходной матрицы А и обратной матрицы А -1 .

В результате умножения матриц получилась единичная матрица. Следовательно, вычисления произведены правильно.

Ответ:

Решение матричных уравнений

Матричные уравнения могут иметь вид:

АХ = В, ХА = В, АХВ = С,

где А,В,С — задаваемые матрицы, Х- искомая матрица.

Матричные уравнения решаются с помощью умножения уравнения на обратные матрицы.

Например, чтобы найти матрицу из уравнения , необходимо умножить это уравнение на слева.

Следовательно, чтобы найти решение уравнения , нужно найти обратную матрицу и умножить ее на матрицу , стоящие в правой части уравнения.

Аналогично решаются другие уравнения.

Пример 2

Решить уравнение АХ = В, если

Решение : Так как обратная матрица равняется (см. пример 1)

Матричный метод в экономическом анализе

Наряду с другими в находят применение также матричные методы . Эти методы базируются на линейной и векторно-матричной алгебре. Такие методы применяются для целей анализа сложных и многомерных экономических явлений. Чаще всего эти методы используются при необходимости сравнительной оценки функционирования организаций и их структурных подразделений.

В процессе применения матричных методов анализа можно выделить несколько этапов.

На первом этапе осуществляется формирование системы экономических показателей и на ее основе составляется матрица исходных данных , которая представляет собой таблицу, в которой по ее отдельным строкам показываются номера систем (i = 1,2,….,n) , а по вертикальным графам — номера показателей (j = 1,2,….,m) .

На втором этапе по каждой вертикальной графе выявляется наибольшее из имеющихся значений показателей, которое и принимается за единицу.

После этого все суммы, отраженные в данной графе делят на наибольшее значение и формируется матрица стандартизированных коэффициентов .

На третьем этапе все составные части матрицы возводят в квадрат. Если они имеют различную значимость, то каждому показателю матрицы присваивается определенный весовой коэффициент k . Величина последнего определяется экспертным путем.

На последнем, четвертом этапе найденные величины рейтинговых оценок R j группируются в порядке их увеличения или уменьшения.

Изложенные матричные методы следует использовать, например, при сравнительном анализе различных инвестиционных проектов, а также при оценке других экономических показателей деятельности организаций.

Данный онлайн калькулятор решает систему линейных уравнений матричным методом. Дается очень подробное решение. Для решения системы линейных уравнений выберите количество переменных. Выбирайте метод вычисления обратной матрицы. Затем введите данные в ячейки и нажимайте на кнопку «Вычислить».

×

Предупреждение

Очистить все ячейки?

Закрыть Очистить

Инструкция ввода данных. Числа вводятся в виде целых чисел (примеры: 487, 5, -7623 и т.д.), десятичных чисел (напр. 67., 102.54 и т.д.) или дробей. Дробь нужно набирать в виде a/b, где a и b целые или десятичные числа. Примеры 45/5, 6.6/76.4, -7/6.7 и т.д.

Матричный метод решения систем линейных уравнений

Рассмотрим следующую систему линейных уравнений:

Учитывая определение обратной матрицы, имеем A −1 A =E , где E — единичная матрица. Следовательно (4) можно записать так:

Таким образом, для решения системы линейных уравнений (1) (или (2)), достаточно умножить обратную к A матрицу на вектор ограничений b .

Примеры решения системы линейных уравнений матричным методом

Пример 1. Решить следующую систему линейных уравнений матричным методом:

Найдем обратную к матрице A методом Жордана-Гаусса. С правой стороны матрицы A запишем единичную матрицу:

Исключим элементы 1-го столбца матрицы ниже главной диагонали. Для этого сложим строки 2,3 со строкой 1, умноженной на -1/3,-1/3 соответственно:

Исключим элементы 2-го столбца матрицы ниже главной диагонали. Для этого сложим строку 3 со строкой 2, умноженной на -24/51:

Исключим элементы 2-го столбца матрицы выше главной диагонали. Для этого сложим строку 1 со строкой 2, умноженной на -3/17:

Отделяем правую часть матрицы. Полученная матрица является обратной матрицей к A :

Матричный вид записи системы линейных уравнений: Ax=b , где

Вычислим все алгебраические дополнения матрицы A :

,
,
,
,
,
,
,
,
.

Обратная матрица вычисляется из следующего выражения.

Применение уравнений широко распространено в нашей жизни. Они используются во многих расчетах, строительстве сооружений и даже спорте. Уравнения человек использовал еще в древности и с тех пор их применение только возрастает. Матричный метод позволяет находить решения СЛАУ (система линейных алгебраических уравнений) любой сложности. Весь процесс решения СЛАУ сводится к двум основным действиям:

Определение обратной матрицы на основании главной матрицы:

Умножение полученной обратной матрицы на вектор-столбец решений.

Допустим, дано СЛАУ следующего вида:

\[\left\{\begin{matrix} 5x_1 + 2x_2 & = & 7 \\ 2x_1 + x_2 & = & 9 \end{matrix}\right.\]

Начнем решение данного уравнения с выписывания матрицы системы:

Матрица правой части:

Определим обратную матрицу. Найти матрицу 2-го порядка можно следующим образом: 1 — сама матрица должна быть невырожденной; 2 — ее элементы, которые находятся на главной диагонали, меняем местами, а у элементов побочной диагонали выполняем смену знака на противоположный, после чего выполняем деление полученных элементов на определитель матрицы. Получим:

\[\begin{pmatrix} 7 \\ 9 \end{pmatrix}=\begin{pmatrix} -11 \\ 31 \end{pmatrix}\Rightarrow \begin{pmatrix} x_1 \\ x_2 \end{pmatrix} =\begin{pmatrix} -11 \\ 31 \end{pmatrix} \]

2 матрицы считаются равными, если равны их соответствующие элементы. В итоге имеем следующий ответ решения СЛАУ:

Где можно решить систему уравнений матричным методом онлайн?

Решить систему уравнений вы можете на нашем сайте . Бесплатный онлайн решатель позволит решить уравнение онлайн любой сложности за считанные секунды. Все, что вам необходимо сделать — это просто ввести свои данные в решателе. Так же вы можете и узнать, как решить уравнение на нашем сайте. А если у вас остались вопросы, то вы можете задать их в нашей групе Вконтакте.

Тема 2. СИСТЕМЫ ЛИНЕЙНЫХ АЛГЕБРАИЧЕСКИХ УРАВНЕНИЙ.

Основные понятия.

Определение 1 . Системой m линейных уравнений с n неизвестными называется система вида:

где и — числа.

Определение 2 . Решением системы (I) называется такой набор неизвестных , при котором каждое уравнение этой системы обращается в тождество.

Определение 3 . Система (I) называется совместной , если она имеет хотя бы одно решение и несовместной , если она не имеет решений. Совместная система называется определенной , если она имеет единственное решение, и неопределенной в противном случае.

Определение 4 . Уравнение вида

называется нулевым , а уравнение вида

называется несовместным . Очевидно, что система уравнений, содержащая несовместное уравнение, является несовместной.

Определение 5 . Две системы линейных уравнений называются равносильными , если каждое решение одной системы служит решением другой и, наоборот, всякое решение второй системы является решением первой.

Матричная запись системы линейных уравнений.

Рассмотрим систему (I) (см. §1).

Обозначим:

Матрица коэффициентов при неизвестных

Матрица – столбец свободных членов

Матрица – столбец неизвестных

.

Определение 1. Матрица называется основной матрицей системы (I), а матрица — расширенной матрицей системы (I).

По определению равенства матриц системе (I) соответствует матричное равенство:

.

Правую часть этого равенства по определению произведения матриц (см. определение 3 § 5 главы 1 ) можно разложить на множители:

, т.е.

Равенство (2) называется матричной записью системы (I) .

Решение системы линейных уравнений методом Крамера.

Пусть в системе (I) (см. §1) m=n , т.е. число уравнений равно числу неизвестных, и основная матрица системы невырожденная, т.е. . Тогда система (I) из §1 имеет единственное решение

где Δ = det A называется главным определителем системы (I), Δ i получается из определителя Δ заменой i -го столбца на столбец из свободных членов системы (I).

Пример.Решить систему методом Крамера:

.

По формулам (3) .

Вычисляем определители системы:

,

,

.

Чтобы получить определитель , мы заменили в определителе первый столбец на столбец из свободных членов; заменяя в определителе 2-ой столбец на столбец из свободных членов, получаем ; аналогичным образом, заменяя в определителе 3-ий столбец на столбец из свободных членов, получаем . Решение системы:

Решение систем линейных уравнений с помощью обратной матрицы.

Пусть в системе(I) (см. §1) m=n и основная матрица системы невырожденная . Запишем систему (I) в матричном виде (см. §2 ):

т.к. матрица A невырожденная, то она имеет обратную матрицу (см. теорему 1 §6 главы 1 ). Умножим обе части равенства (2) на матрицу , тогда

По определению обратной матрицы . Из равенства (3) имеем

Решить систему с помощью обратной матрицы

.

Обозначим

В примере (§ 3)мы вычислили определитель , следовательно, матрица A имеет обратную матрицу . Тогда в силу (4) , т.е.

. (5)

Найдем матрицу (см. §6 главы 1 )

, , ,

, , ,

,

.

Метод Гаусса.

Пусть задана система линейных уравнений:

. (I)

Требуется найти все решения системы (I) или убедиться в том, что система несовместна.

Определение 1. Назовем элементарным преобразованием системы (I) любое из трёх действий:

1) вычёркивание нулевого уравнения;

2) прибавление к обеим частям уравнения соответствующих частей другого уравнения, умноженных на число l;

3) перемена местами слагаемых в уравнениях системы так, чтобы неизвестные с одинаковыми номерами во всех уравнениях занимали одинаковые места, т.е. если, например, в 1-ом уравнении мы поменяли 2-ое и 3-е слагаемые, тогда то же самое необходимо сделать во всех уравнениях системы.

Метод Гаусса состоит в том, что система (I) с помощью элементарных преобразований приводится к равносильной системе, решение которой находится непосредственно или устанавливается её неразрешимость.

Как было описано в §2 система (I) однозначно определяется своей расширенной матрицей и любое элементарное преобразование системы (I) соответствует элементарному преобразованию расширенной матрицы:

.

Преобразование 1) соответствует вычёркиванию нулевой строки в матрице , преобразование 2) равносильно прибавлению к соответствующей строке матрицы другой её строки, умноженной на число l, преобразование 3) эквивалентно перестановке столбцов в матрице .

Легко видеть, что, наоборот, каждому элементарному преобразованию матрицы соответствует элементарное преобразование системы (I). В силу сказанного, вместо операций с системой (I) мы будем работать с расширенной матрицей этой системы.

В матрице 1-ый столбец состоит из коэффициентов при х 1 , 2-ой столбец — из коэффициентов при х 2 и т.д. В случае перестановки столбцов следует учитывать, что это условие нарушается. Например, если мы поменяем 1-ый и 2-ой столбцы местами, то теперь в 1-ом столбце будут коэффициенты при х 2 , а во 2-ом столбце — коэффициенты при х 1 .

Будем решать систему (I) методом Гаусса.

1. Вычеркнем в матрице все нулевые строки, если такие имеются (т.е. вычеркнем в системе (I) все нулевые уравнения).

2. Проверим, есть ли среди строк матрицы строка, в которой все элементы, кроме последнего, равны нулю (назовём такую строку несовместной). Очевидно, что такой строке соответствует несовместное уравнение в системе (I) , следовательно, система (I) решений не имеет и на этом процесс заканчивается.

3. Пусть матрица не содержит несовместных строк (система (I) не содержит несовместных уравнений). Если a 11 =0 , то находим в 1-ой строке какой-нибудь элемент (кроме последнего) отличный от нуля и переставляем столбцы так, чтобы в 1-ой строке на 1-ом месте не было нуля. Будем теперь считать, что (т.е. поменяем местами соответствующие слагаемые в уравнениях системы (I)).

4. Умножим 1-ую строку на и сложим результат со 2-ой строкой, затем умножим 1-ую строку на и сложим результат с 3-ей строкой и т.д. Очевидно, что этот процесс эквивалентен исключению неизвестного x 1 из всех уравнений системы (I), кроме 1-ого. В новой матрице получаем нули в 1-ом столбце под элементом a 11 :

.

5. Вычеркнем в матрице все нулевые строки, если они есть, проверим, нет ли несовместной строки (если она имеется, то система несовместна и на этом решение заканчивается). Проверим, будет ли a 22 / =0 , если да, то находим во 2-ой строке элемент, отличный от нуля и переставляем столбцы так, чтобы . Далее умножаем элементы 2-ой строки на и складываем с соответствующими элементами 3-ей строки, затем — элементы 2-ой строки на и складываем с соответствующими элементами 4-ой строки и т.д., пока не получим нули под a 22 /

.

Произведенные действия эквивалентны исключению неизвестного х 2 из всех уравнений системы (I), кроме 1-ого и 2-ого. Так как число строк конечно, поэтому через конечное число шагов мы получим, что либо система несовместна, либо мы придём к ступенчатой матрице (см. определение 2 §7 главы 1 ) :

,

Выпишем систему уравнений, соответствующую матрице . Эта система равносильна системе (I)

.

Из последнего уравнения выражаем ; подставляем в предыдущее уравнение, находим и т.д., пока не получим .

Замечание 1. Таким образом, при решении системы (I) методом Гаусса мы приходим к одному из следующих случаев.

1. Система (I) несовместна.

2. Система (I) имеет единственное решение, если в матрице число строк равно числу неизвестных ().

3. Система (I) имеет бесчисленное множество решений, если число строк в матрице меньше числа неизвестных ().

Отсюда имеет место следующая теорема.

Теорема. Система линейных уравнений либо несовместна, либо имеет единственное решение, либо – бесконечное множество решений.

Примеры. Решить систему уравнений методом Гаусса или доказать ее несовместность:

б) ;

а) Перепишем заданную систему в виде:

.

Мы поменяли местами 1-ое и 2-ое уравнение исходной системы, чтобы упростить вычисления (вместо дробей мы с помощью такой перестановки будем оперировать только целыми числами).

Составляем расширенную матрицу:

.

Нулевых строк нет; несовместных строк нет, ; исключим 1-ое неизвестное из всех уравнений системы, кроме 1-го. Для этого умножим элементы 1-ой строки матрицы на «-2» и сложим с соответствующими элементами 2-ой строки, что равносильно умножению 1-го уравнения на «-2» и сложению со 2-ым уравнением. Затем умножим элементы 1-ой строки на «-3» и сложим с соответствующими элементами третьей строки, т.е. умножим 2-ое уравнение заданной системы на «-3» и сложим с 3-им уравнением. Получим

.

Матрице соответствует система уравнений). — (см. определение 3§7 главы 1).

4.6: Решение систем уравнений с использованием матриц

  1. Последнее обновление
  2. Сохранить как PDF
  • Идентификатор страницы
    5142
    • OpenStax
    • OpenStax
    Цели обучения

    К концу этого раздела вы сможете:

    • Записывать расширенную матрицу для системы уравнений
    • Использовать операции со строками на матрице
    • Решение систем уравнений с использованием матриц

    Прежде чем приступить к работе, пройдите этот тест на готовность.

    1. Решить: \(3(x+2)+4=4(2x−1)+9\).
      Если вы пропустили эту проблему, просмотрите [ссылка] .
    2. Решите: \(0,25p+0,25(x+4)=5,20\).
      Если вы пропустили эту проблему, просмотрите 92\).
      Если вы пропустили эту проблему, просмотрите [ссылка] .

    Напишите расширенную матрицу для системы уравнений

    Решение системы уравнений может быть утомительной операцией, где простая ошибка может нанести ущерб поиску решения. Доступен альтернативный метод, использующий основные процедуры исключения, но с более простыми обозначениями. Метод включает использование матрицы . Матрица представляет собой прямоугольный массив чисел, расположенных в строках и столбцах.

    МАТРИЦА

    Матрица представляет собой прямоугольный массив чисел, расположенных в строках и столбцах.

    Матрица с m строк и n столбцов имеет порядок \(m\times n\). Матрица слева внизу имеет 2 строки и 3 столбца, поэтому она имеет порядок \(2\times 3\). Мы говорим, что это матрица 2 на 3.

    Каждое число в матрице называется элементом или записью в матрице.

    Мы будем использовать матрицу для представления системы линейных уравнений. Мы записываем каждое уравнение в стандартной форме, а коэффициенты переменных и константы каждого уравнения становятся строкой в ​​матрице. Тогда каждый столбец будет коэффициентом одной из переменных в системе или констант. Вертикальная черта заменяет знаки равенства. Полученную матрицу назовем расширенной матрицей системы уравнений.

    Обратите внимание, что первый столбец состоит из всех коэффициентов x , второй столбец содержит все коэффициенты y , а третий столбец содержит все константы.

    Пример \(\PageIndex{1}\)

    ⓐ \(\left\{ \begin{array} {l} 5x−3y=−1 \\ y=2x−2 \end{array} \right. \) ⓑ \( \left\{ \begin{array} {l} 6x−5y+2z=3 \\ 2x+y−4z=5 \\ 3x−3y+z=−1 \end{array} \right . \)

    Ответ

    ⓐ Второе уравнение не в стандартной форме. Перепишем второе уравнение в стандартной форме.

    \[\begin{align} y=2x−2 \\ −2x+y=−2 \end{align} \nonumber\]

    Заменим второе уравнение его стандартной формой. В расширенной матрице первое уравнение дает нам первую строку, а второе уравнение дает нам вторую строку. Вертикальная черта заменяет знаки равенства.

    ⓑ Все три уравнения в стандартной форме. В расширенной матрице первое уравнение дает нам первую строку, второе уравнение дает нам вторую строку, а третье уравнение дает нам третью строку. Вертикальная черта заменяет знаки равенства.

    Пример \(\PageIndex{2}\)

    Запишите каждую систему линейных уравнений в виде расширенной матрицы:

    ⓐ \(\left\{ \begin{array} {l} 3x+8y=−3 \\ 2x=−5y−3 \end{array} \right. \) ⓑ \(\left\{ \begin{array} {l} 2x−5y+3z=8 \\ 3x−y+4z=7 \\ x +3y+2z=−3 \end{массив} \right. \)

    Ответ

    ⓐ \(\left[ \begin{matrix} 3 &8 &-3 \\ 2 &5 &−3 \end{matrix} \right] \)

    ⓑ \(\left[ \begin{matrix} 2 &3 &1 &−5 \\ ​​−1 &3 &3 &4 \\ 2 &8 &7 &−3 \end{matrix} \right] \)

    Пример \(\PageIndex{3}\)

    Запишите каждую систему линейных уравнений в виде расширенной матрицы:

    ⓐ \(\left\{ \begin{array} {l} 11x=−9y−5 \\ ​​7x +5y=−1 \end{array} \right.\) ⓑ \(\left\{ \begin{array} {l} 5x−3y+2z=−5 \\ ​​2x−y−z=4 \\ 3x −2y+2z=−7 \end{массив} \right.\)

    Ответ

    ⓐ \(\left[ \begin{matrix} 11 &9 &−5 \\ ​​7 &5 &−1 \end{matrix} \right] \)
    ⓑ \(\left[ \begin{matrix} 5 &−3 &2 &−5 \\ ​​2 &−1 &−1 &4 \\ 3 &−2 &2 &−7 \end{matrix} \right] \)

    При решении систем уравнений с использованием матриц важно иметь возможность переключаться между системой и матрицей. В следующем примере нас просят взять информацию из матрицы и написать систему уравнений.

    Пример \(\PageIndex{4}\)

    Запишите систему уравнений, соответствующую расширенной матрице:

    \(\left[ \begin{array} {ccc|c} 4 &−3 &3 &− 1 \\ 1 &2 &−1 &2 \\ −2 &−1 &3 &−4 \end{массив} \right] \).

    Ответить

    Мы помним, что каждая строка соответствует уравнению и что каждая запись является коэффициентом переменной или константой. Вертикальная черта заменяет знак равенства. Поскольку эта матрица представляет собой \(4\times 3\), мы знаем, что она преобразуется в систему трех уравнений с тремя переменными.

    Пример \(\PageIndex{5}\)

    Запишите систему уравнений, соответствующую расширенной матрице: \(\left[ \begin{matrix} 1 &−1 &2 &3 \\ 2 &1 &−2 &1 \\ 4 &−1 &2 &0 \end{matrix} \right] \).

    Ответить

    \(\left\{ \begin{array} {l} x−y+2z=3 \\ 2x+y−2z=1 \\ 4x−y+2z=0 \end{array} \right.\)

    Пример \(\PageIndex{6}\)

    Напишите систему уравнений, соответствующую расширенной матрице: \(\left[ \begin{matrix} 1 &1 &1 &4 \\ 2 &3 &−1 &8 \\ 1 &1 &−1 &3 \end{matrix} \ Правильно] \).

    Ответить

    \(\left\{ \begin{array} {l} x+y+z=4 \\ 2x+3y−z=8 \\ x+y-z=3 \end{array} \right.\)

    Использование операций со строками над матрицей

    После того, как система уравнений будет представлена ​​в расширенной матричной форме, мы будем выполнять операции над строками, которые приведут нас к решению.

    Для решения методом исключения не имеет значения, в каком порядке мы располагаем уравнения в системе. Точно так же в матрице мы можем поменять местами строки.

    При решении методом исключения мы часто умножаем одно из уравнений на константу. Поскольку каждая строка представляет собой уравнение, и мы можем умножить каждую часть уравнения на константу, аналогичным образом мы можем умножить каждую запись в строке на любое действительное число, кроме 0.

    При исключении мы часто добавляем кратное одной строке к другой ряд. В матрице мы можем заменить строку на ее сумму, кратную другой строке.

    Эти действия называются операциями со строками и помогут нам использовать матрицу для решения системы уравнений.

    ОПЕРАЦИИ СО СТРОКАМИ

    В матрице следующие операции могут выполняться над любой строкой, и результирующая матрица будет эквивалентна исходной матрице.

    1. Поменяйте местами любые два ряда.
    2. Умножить строку на любое действительное число, кроме 0.
    3. Добавить ненулевое кратное одной строки к другой строке.

    Выполнение этих операций несложно, но все арифметические действия могут привести к ошибке. Если мы используем систему для записи операций со строками на каждом этапе, гораздо проще вернуться и проверить нашу работу.

    Мы используем заглавные буквы с нижними индексами для представления каждой строки. Затем мы показываем операцию слева от новой матрицы. Чтобы показать перестановку строк:

    Чтобы умножить строку 2 на \(−3\):

    Чтобы умножить строку 2 на \(−3\) и добавить ее к строке 1:

    Пример \ (\PageIndex{7}\)

    Выполнить указанные операции над расширенной матрицей:

    ⓐ Поменять местами строки 2 и 3.

    ⓑ Умножить строку 2 на 5.

    ⓒ Умножить строку 3 на −2−2 и прибавить к строке 1.

    \( \left[ \begin{array} {ccc|c} 6 &−5 &2 &3 \\ 2 &1 &−4 &5 \\ 3 &−3 &1 &−1 \end{массив} \right] \)

    Ответ

    ⓐ Меняем местами 2 и 3 ряды.

    ⓑ Умножаем 2 ряд на 5.

    ⓒ Умножаем строку 3 на \(−2\) и прибавляем к строке 1.

    Пример \(\PageIndex{8}\)

    Выполнить указанные операции над расширенной матрицей:

    ⓐ Поменять местами строки 1 и 3.

    ⓑ Умножить строку 3 на 3.

    ⓒ Умножить строку 3 на 2 и прибавить к строке 2.

    \( \left[ \begin{array} {ccc|c} 5 &−2 &-2 &-2 \\ 4 &-1 &−4 &4 \\ -2 &3 &0 &−1 \end{массив} \right] \)

    Ответ

    ⓐ \( \left[ \begin{matrix} −2 &3 &0 &−2 \\ 4 &−1 &−4 &4 \\ 5 &−2 &−2 &−2 \end{matrix} \right] \ )

    ⓑ \( \left[ \begin{matrix} −2 &3 &0 &−2 \\ 4 &−1 &−4 &4 \\ 15 &−6 &−6 &−6 \end{matrix} \right] \ )

    ⓒ \( \left[ \begin{matrix} -2 &3 &0 &2 & \\ 3 &4 &-13 &-16 &-8 \\ 15 &-6 &-6 &-6 & \end{matrix} \ справа] \)

    Пример \(\PageIndex{9}\)

    Выполнить указанные операции над расширенной матрицей:

    ⓐ Поменять местами строки 1 и 2,

    ⓑ Умножить строку 1 на 2,

    ⓒ Умножить строку 2 на 3 и сложить в строку 1.

    \( \left[ \begin{array} {ccc|c} 2 &−3 &−2 &−4 \\ 4 &1 &−3 &2 \\ 5 &0 &4 &−1 \end{ массив} \справа] \)

    Ответить

    ⓐ \( \left[ \begin{matrix} 4 &1 &−3 &2 \\ 2 &−3 &−2 &−4 \\ 5 &0 &4 &−1 \end{matrix} \right] \)
    ⓑ \( \left[ \begin{matrix} 8 &2 &−6 &4 \\ 2 &−3 &−2 &−4 \\ 5 &0 &4 &−1 \end{matrix} \right] \)
    ⓒ \( \left[ \begin{matrix} 14 &−7 &−12 &−8 \\ 2 &−3 &−2 &−4 \\ 5 &0 &4 &−1 \end{matrix} \right] \)

    Теперь, когда мы попрактиковались в операциях со строками, мы рассмотрим расширенную матрицу и выясним, какую операцию мы будем использовать для достижения цели. Это именно то, что мы сделали, когда мы сделали исключение. Мы решили, на какое число умножить строку, чтобы при сложении строк исключалась переменная.

    Учитывая эту систему, что бы вы сделали, чтобы исключить x ?

    Следующий пример делает то же самое, но с матрицей.

    Пример \(\PageIndex{10}\)

    Выполните необходимую операцию со строками, чтобы первая запись в строке 2 была равна нулю в расширенной матрице: \( \left[ \begin{array} {cc|c } 1 &−1 &2 \\ 4 &−8 &0 \end{массив} \right] \)

    Ответ

    Чтобы число 4 стало равным 0, мы могли бы умножить строку 1 на \(−4\), а затем прибавить ее к строке 2.

    Пример \(\PageIndex{11}\)

    Выполните необходимую операцию со строками, чтобы первая запись в строке 2 была равна нулю в расширенной матрице: \( \left[ \begin{array} {cc|c} 1 &−1 &2 \\ 3 &−6 &2 \end{массив} \right] \)

    Ответить

    \( \left[ \begin{matrix} 1 &−1 &2 \\ 0 &−3 &−4 \end{matrix} \right] \)

    Пример \(\PageIndex{12}\)

    Выполните необходимую операцию со строками, чтобы первая запись в строке 2 была равна нулю в расширенной матрице: \( \left[ \begin{array} {cc|c} 1 &−1 &3 \\ -2 &−3 &2 \end{массив} \right] \)

    Ответ

    \( \left[ \begin{matrix} 1 &−1 &3 \\ 0 &−5 &8 \end{matrix} \right] \)

    Решение систем уравнений с использованием матриц

    Чтобы решить систему уравнений с использованием матриц, мы преобразуем расширенную матрицу в матрицу строк-ступенчатой ​​формы , используя операции со строками. Для непротиворечивой и независимой системы уравнений ее расширенная матрица имеет форму эшелона строк, когда слева от вертикальной линии каждая запись на диагонали равна 1, а все записи ниже диагонали — нули.

    РЯД-ЭШЕЛОН ФОРМА

    Для непротиворечивой и независимой системы уравнений ее расширенная матрица имеет вид строк-ступенчатой ​​формы , когда слева от вертикальной линии каждая запись на диагонали равна 1, а все записи ниже диагонали — нулями.

    Как только мы приведем расширенную матрицу к ступенчатой ​​форме, мы можем написать эквивалентную систему уравнений и прочитать значение хотя бы одной переменной. Затем мы подставляем это значение в другое уравнение, чтобы продолжить решение для других переменных. Этот процесс проиллюстрирован в следующем примере.

    Как решить систему уравнений с помощью матрицы

    Решить систему уравнений с помощью матрицы: \(\left\{ \begin{array} {l} 3x+4y=5 \\ x+2y=1 \ end{массив} \right. \)

    Ответ

    Пример \(\PageIndex{14}\)

    Решить систему уравнений с помощью матрицы: \(\left\{ \begin{array} {l} 2x+y=7 \\ x−2y=6 \end {массив} \право.\)

    Ответ

    Решение: \((4,−1)\).

    Пример \(\PageIndex{15}\)

    Решить систему уравнений с помощью матрицы: \(\left\{ \begin{array} {l} 2x+y=−4 \\ x−y=−2 \end{массив} \right.\)

    Ответ

    Решение: \((−2,0)\).

    Здесь перечислены шаги.

    РЕШИТЬ СИСТЕМУ УРАВНЕНИЙ С ИСПОЛЬЗОВАНИЕМ МАТРИЦ.
    1. Напишите расширенную матрицу для системы уравнений.
    2. С помощью операций со строками значение записи в строке 1 столбца 1 равно 1.
    3. Используя операции со строками, получить нули в столбце 1 ниже 1.
    4. Используя операции со строками, сделайте запись в строке 2 столбца 2 равной 1.
    5. Продолжайте процесс до тех пор, пока матрица не будет иметь форму строки-эшелона.
    6. Напишите соответствующую систему уравнений.
    7. Используйте подстановку, чтобы найти оставшиеся переменные.
    8. Запишите решение в виде упорядоченной пары или тройки.
    9. Убедитесь, что решение соответствует исходным уравнениям.

    Вот изображение, показывающее порядок получения 1 и 0 в правильном положении для формы строки-эшелона.

    Мы используем ту же процедуру, когда система уравнений состоит из трех уравнений.

    Пример \(\PageIndex{16}\)

    Решите систему уравнений с помощью матрицы: \(\left\{ \begin{array} {l} 3x+8y+2z=−5 \\ ​​2x+5y −3z=0 \\ x+2y−2z=−1 \end{массив} \right.\)

    Ответить
    Write the augmented matrix for the equations. Row 1 is 3, 8, 2, minus 5. Row 2 is 2, 5, minus 3, 0. Row 3 is 1, 2, minus 2, minus 1. Interchange row 1 and 3 to get the entry in row 1, column 1 to be 1. Use operation minus 2R1 plus R2 on row 2. Use operation minus 3R1 plus R3 on row 3. Use operation minus 2R2 plus R3 on row 3. Use operation 1 upon 6 R3 on row 3. The matrix is now in row-echelon form. The corresponding system of equations is x plus 2y minus 2z equals minus 1, y plus z equals 2 and z equals minus 1. Using substitution, we get y equal to 3 and x equal to minus 9. The solution is minus 9, 3, minus 1. Check that the original equations hold true.»>
     
    Напишите расширенную матрицу для уравнений.
    Поменяйте местами строки 1 и 3, чтобы получить запись в
    строке 1, столбце 1, равной 1.
    Используя операции со строками, получить нули в столбце 1 под 1.
     
    Запись в строке 2 столбца 2 теперь равна 1.  
    Продолжайте процесс до тех пор, пока матрица
    не будет иметь форму строки-эшелона.
     
    Матрица теперь имеет форму строки-эшелона.
    Напишите соответствующую систему уравнений.
    Используйте подстановку, чтобы найти остальные переменные.
     
    Запишите решение в виде упорядоченной пары или тройки.
    Убедитесь, что решение соответствует исходным уравнениям. Мы оставляем вам чек.
    Пример \(\PageIndex{17}\)

    Решите систему уравнений с помощью матрицы: \(\left\{ \begin{array} {l} 2x−5y+3z=8 \\ 3x−y+4z =7 \\ x+3y+2z=−3 \end{массив} \right. \)

    Ответ

    \((6,−1,−3)\)

    Пример \(\PageIndex{18}\)

    Решить систему уравнений с помощью матрицы: \(\left\{ \begin{array} {l} −3x+y+z=−4 \\ −x+ 2y−2z=1 \\ 2x−y−z=−1 \end{массив} \right.\)

    Ответить

    \((5,7,4)\)

    До сих пор мы работали с матрицами только с непротиворечивыми и независимыми системами, что означает, что они имеют ровно одно решение. Давайте теперь посмотрим, что происходит, когда мы используем матрицу для зависимой или противоречивой системы.

    Пример \(\PageIndex{19}\)

    Решите систему уравнений с помощью матрицы: \(\left\{ \begin{array} {l} x+y+3z=0 \\ x+3y+ 5z=0 \\ 2x+4z=1 \end{массив} \right. \)

    Ответить
     
    Напишите расширенную матрицу для уравнений.
    Запись в строке 1, столбце 1 равна 1.  
    Используя операции со строками, получить нули в столбце 1 под 1.
     
    Продолжайте процесс до тех пор, пока матрица не будет иметь форму строки-эшелона.
    Умножьте строку 2 на 2 и добавьте к строке 3.
    На данный момент у нас есть все нули слева от строки 3.  
    Напишите соответствующую систему уравнений.
    Так как \(0 \neq 1 \) мы имеем ложное утверждение. Точно так же, как когда мы решали систему, используя другие методы, это говорит нам о несогласованности системы. Нет решения.
    Пример \(\PageIndex{20}\)

    Решить систему уравнений с помощью матрицы: \(\left\{ \begin{array} {l} x−2y+2z=1 \\ −2x+y− z=2 \\ x−y+z=5 \end{array} \right.\)

    Ответ

    нет решения

    Пример \(\PageIndex{21}\)

    Решить систему уравнений с помощью матрицы: \(\left\{ \begin{array} {l} 3x+4y−3z=−2 \\ −2x+3y −z=−1 \\ 2x+y−2z=6 \end{массив} \right. \)

    Ответ

    нет решения

    Последняя система была несовместимой и поэтому не имела решений. Следующий пример является зависимым и имеет бесконечно много решений.

    Пример \(\PageIndex{22}\)

    Решить систему уравнений с помощью матрицы: \(\left\{ \begin{array} {l} x−2y+3z=1 \\ x+y−3z =7 \\ 3x−4y+5z=7 \end{массив} \right. \)

    Ответ
     
    Напишите расширенную матрицу для уравнений.
    Запись в строке 1, столбце 1 равна 1.  
    Используя операции со строками, получить нули в столбце 1 под 1.
     
    Продолжайте процесс до тех пор, пока матрица не будет иметь форму строки-эшелона.
    Умножьте строку 2 на \(−2\) и добавьте ее к строке 3.
    На данный момент у нас есть все нули в нижней строке.  
    Напишите соответствующую систему уравнений.
    Так как \(0=0\) мы имеем истинное утверждение. Так же, как когда мы решали подстановкой, это говорит нам о том, что у нас есть зависимая система. Существует бесконечно много решений.
    Найдите y через z во втором уравнении.
    Решите первое уравнение для x через z .
    Замените \(y=2z+2\).
    Упрощение.
    Упрощение.
    Упрощение.
    Система имеет бесконечно много решений \((x,y,z)\), где \(x=z+5;\space y=2z+2;\space z\) — любое действительное число.
    Пример \(\PageIndex{23}\)

    Решить систему уравнений с помощью матрицы: \(\left\{ \begin{array} {l} x+y−z=0 \\ 2x+4y−2z =6 \\ 3x+6y−3z=9 \end{массив} \right. \)

    Ответ

    бесконечно много решений \((x,y,z)\), где \(x=z−3;\space y=3;\space z\) — любое действительное число.

    Пример \(\PageIndex{24}\)

    Решить систему уравнений с помощью матрицы: \(\left\{ \begin{array} {l} x−y−z=1 \\ −x+2y− 3z=−4 \\ 3x−2y−7z=0 \end{array} \right. \)

    Ответить

    бесконечно много решений \((x,y,z)\), где \(x=5z−2;\space y=4z−3;\space z\) — любое действительное число.

    Получите доступ к этому онлайн-ресурсу для получения дополнительных инструкций и практики с исключением Гаусса.

    • Исключение Гаусса

    Основные понятия

    • Матрица: Матрица представляет собой прямоугольный массив чисел, расположенных в строках и столбцах. Матрица с m строк и n столбцов имеют порядка \(m\times n\). Матрица слева внизу имеет 2 строки и 3 столбца, поэтому она имеет порядок \(2\times 3\). Мы говорим, что это матрица 2 на 3.

      Каждое число в матрице называется элементом или элементом в матрице.
    • Операции со строками: В матрице следующие операции могут выполняться над любой строкой, и результирующая матрица будет эквивалентна исходной матрице.
      • Поменять местами любые два ряда
      • Умножить строку на любое действительное число, кроме 0
      • Добавить ненулевое кратное одной строки к другой строке
    • Строко-Эшелонная Форма: Для непротиворечивой и независимой системы уравнений ее расширенная матрица имеет форму ступенчато-строки, когда слева от вертикальной линии каждая запись на диагонали равна 1, а все записи ниже диагонали равны нули.
    • Как решить систему уравнений с помощью матриц.
      1. Напишите расширенную матрицу для системы уравнений.
      2. С помощью операций со строками значение записи в строке 1 столбца 1 равно 1.
      3. Используя операции со строками, получить нули в столбце 1 ниже 1.
      4. Используя операции со строками, сделайте запись в строке 2 столбца 2 равной 1.
      5. Продолжайте процесс до тех пор, пока матрица не будет иметь форму строки-эшелона.
      6. Напишите соответствующую систему уравнений.
      7. Используйте подстановку, чтобы найти оставшиеся переменные.
      8. Запишите решение в виде упорядоченной пары или тройки.
      9. Убедитесь, что решение соответствует исходным уравнениям.

    Глоссарий

    Матрица
    Матрица представляет собой прямоугольный массив чисел, расположенных в строках и столбцах.
    рядно-эшелонная форма
    Матрица представляет собой ступенчатую матрицу, когда слева от вертикальной линии каждый элемент по диагонали равен 1, а все элементы под диагональю равны нулю.

    Эта страница под названием 4.6: Решение систем уравнений с использованием матриц распространяется под лицензией CC BY 4.0 и была создана, изменена и/или курирована OpenStax с использованием исходного контента, который был отредактирован в соответствии со стилем и стандартами платформы LibreTexts; подробная история редактирования доступна по запросу.

    1. Наверх
      • Была ли эта статья полезной?
      1. Тип изделия
        Раздел или Страница
        Автор
        ОпенСтакс
        Лицензия
        СС BY
        Версия лицензии
        4,0
        Программа OER или Publisher
        ОпенСтакс
        Показать страницу TOC
        нет
      2. Теги
        1. источник@https://openstax. org/details/books/intermediate-алгебра-2e

      Решение матричных уравнений

      Горячая математика

      А матричное уравнение уравнение, в котором переменная обозначает матрица .

      Вы можете решить более простые матричные уравнения, используя сложение матриц а также скалярное умножение .

      Примеры 1:

      Решите для матрицы Икс : Икс + [ 3 2 1 0 ] знак равно [ 6 3 7 − 1 ]

      Икс + [ 3 2 1 0 ] − [ 3 2 1 0 ] знак равно [ 6 3 7 − 1 ] − [ 3 2 1 0 ] Икс + [ 0 0 0 0 ] знак равно [ 6 − 3 3 − 2 7 − 1 − 1 − 0 ] Икс знак равно [ 3 1 6 − 1 ]

      Примеры 2:

      Решите для матрицы Икс : Икс − [ − 9 − 3 6 0 ] знак равно [ 4 0 12 − 10 ]

      Икс − [ − 9 − 3 6 0 ] знак равно [ 4 0 12 − 10 ] Икс − [ − 9− 3 6 0 ] + [ − 9 − 3 6 0 ] знак равно [ 4 0 12 − 10 ] + [ − 9− 3 6 0 ] Икс − [ 0 0 0 0 ] знак равно [ 4 + ( − 9) 0 + ( − 3 ) 12 + 6 − 10 + 0 ] Икс знак равно [ − 5 − 3 18 − 10 ]

      Матричные уравнения можно использовать для решать системы линейных уравнений используя левую и правую части уравнений.

      Примеры 3:

      Решите систему уравнений с помощью матриц: { 7 Икс + 5 у знак равно 3 3 Икс − 2 у знак равно 22

      7 Икс + 5 у знак равно 3 3 Икс − 2 у знак равно 22 → [ 7 Икс + 5 у 3 Икс − 2 у ] знак равно [ 3 22 ]

      Запишите матрицу слева как произведение коэффициентов и переменных.

      [ 7 5 3 − 2 ] [ Икс у ] знак равно [ 3 22 ]

      ↑ ↑ ↑

      коэффициент переменная постоянный матрица матрица матрица

      Сначала найдите обратную матрицу коэффициентов. Обратное [ 7 5 3 − 2 ] является

      1 7 ( − 2 ) − ( 3 ) ( 5 ) [ − 2 − 5 − 3 7 ] знак равно − 1 29[ − 2 − 5 − 3 7 ] знак равно [ 2 29 5 29 3 29 − 7 29]

      Затем умножьте каждую часть матричного уравнения на обратная матрица . Так как умножение матриц нет коммутативной, обратная матрица должна быть слева на каждый часть матричного уравнения.

      [ 2 29 5 29 3 29− 7 29 ] [ 7 5 3 − 2 ] [ Икс у ] знак равно [ 2 29 5 29 3 29− 7 29 ] [ 3 22 ]

      [ 1 0 0 1 ] [ Икс у ] знак равно [ 4 − 5 ]

      единичная матрица слева подтверждает, что обратная матрица была вычислена правильно.

      [ Икс у ] знак равно [ 4 − 5 ]

      Решение ( 4 , − 5 ) .

      Загрузите наши бесплатные приложения для обучения и книги для подготовки к экзаменам

      Матричные уравнения

      Цели
      1. Понимать эквивалентность между системой линейных уравнений, расширенной матрицей, векторным уравнением и матричным уравнением.
      2. Охарактеризовать векторы b так, что Ax=b непротиворечиво, с точки зрения диапазона столбцов A.
      3. Охарактеризуйте матрицы A таким образом, что Ax=b непротиворечиво для всех векторов b.
      4. Рецепт: умножить вектор на матрицу (два способа).
      5. Picture: набор всех векторов b таких, что Ax=b, непротиворечив.
      6. Словарное слово: матричное уравнение .

      В этом разделе мы представляем очень лаконичный способ записи системы линейных уравнений: Ax=b. Здесь A — матрица, а x,b — векторы (обычно разного размера), поэтому сначала мы должны объяснить, как умножать матрицу на вектор.

      Когда мы говорим «A — матрица размера m × n», мы имеем в виду, что A имеет m строк и n столбцов.

      Определение

      Пусть A — матрица размера m×n со столбцами v1,v2,…,vn:

      А=С|||v1v2···vn|||D

      Произведение оператора A с вектором x в Rn является линейной комбинацией

      Ax=C|||v1v2···vn|||DEIIGx1x2…xnFJJH=x1v1+x2v2+···+xnvn.

      Это вектор в Rm.

      Пример

      Чтобы Ax имело смысл, количество элементов x должно совпадать с количеством столбцов A: мы используем элементы x как коэффициенты столбцов A в линейной комбинации. Результирующий вектор имеет то же количество элементов, что и число 9.1089 строк A, так как каждый столбец A имеет такое количество записей.

      Если A представляет собой матрицу размера m × n (m строк, n столбцов), то Ax имеет смысл, когда x имеет n элементов. Произведение Ax содержит m записей.

      Свойства матрично-векторного произведения

      Пусть A — матрица размера m × n, пусть u, v — векторы в Rn, а c — скаляр. Тогда:

      • А(и+в)=Аи+Ав
      • A(cu)=cAu
      Определение

      Матричное уравнение — это уравнение вида Ax=b, где A — матрица размера m × n, b — вектор в Rm, а x — вектор, коэффициенты которого x1,x2,…,xn неизвестны. .

      В этой книге мы изучим два дополнительных вопроса о матричном уравнении Ax=b:

      1. При конкретном выборе b, каковы все решения Ax=b?
      2. Каковы все варианты b, чтобы Ax=b было непротиворечивым?

      Первый вопрос больше похож на вопросы, к которым вы, возможно, уже привыкли из своих предыдущих курсов по алгебре; у вас есть много практики решения уравнений типа x2−1=0 относительно x. Второй вопрос, возможно, является новой концепцией для вас. Теорема о рангах в разделе 2.9, который является кульминацией этой главы, говорит нам, что эти два вопроса тесно связаны.

      Пример

      Мы будем свободно перемещаться между четырьмя способами записи линейной системы снова и снова до конца книги.

      Другой способ вычислений Axe

      Приведенное выше определение является полезным способом определения произведения матрицы на вектор, когда дело доходит до понимания связи между матричными уравнениями и векторными уравнениями. Здесь мы даем определение, которое лучше подходит для ручных вычислений.

      Определение

      Вектор-строка представляет собой матрицу с одной строкой. Произведение вектора-строки длины n и вектора (столбца) длины n равно

      .

      Aa1a2···anBEIIGx1x2…xnFJJH=a1x1+a2x2+···+anxn.

      Это скаляр.

      Рецепт: Правило строки-столбца для умножения матрицы на вектор

      Если A — матрица размера m × n со строками r1,r2,…,rm, а x — вектор в Rn, то

      Ax=EIIG—r1——r2—. ..—rm—FJJHx=EIIGr1xr2x…rmxFJJH.

      Пример

      Пусть A — матрица со столбцами v1,v2,…,vn:

      А=С|||v1v2···vn|||D.

      Затем

      Ax=bhas решение⇐⇒существуют x1,x2,…,xn такие, что AEIIGx1x2…xnFJJH=b⇐⇒существуют x1,x2,…,xn такие, что x1v1+x2v2+···+xnvn=b⇐⇒бисалинейная комбинация v1,v2,… ,vn⇐⇒bis находится в диапазоне столбцов матрицы A.

      Пролеты и согласованность

      Матричное уравнение Ax=b имеет решение тогда и только тогда, когда b находится в промежутке столбцов A.

      Это дает эквивалентность между алгебраический оператор (Ax=b непротиворечив) и геометрический оператор (b находится в диапазоне столбцов A).

      Пример (несогласованная система)
      Пример (согласованная система)
      Когда решения всегда существуют

      Опираясь на это замечание, у нас есть следующий критерий, когда Ax=b соответствует каждому выбору b.

      Теорема

      Пусть A — матрица размера m × n (нерасширенная). Следующие эквивалентны:

      1. Ax=b имеет решение для всех b в Rm.
      2. Длина столбцов A равна Rm.
      3. A имеет точку поворота в каждой строке.
      Доказательство

      Эквивалентность 1 и 2 устанавливается этим примечанием применительно к каждому b в Rm.

      Теперь покажем, что 1 и 3 эквивалентны. (Поскольку мы знаем, что 1 и 2 эквивалентны, отсюда следует, что 2 и 3 также эквивалентны.) Если A имеет центральную точку в каждой строке, то его сокращенная ступенчатая форма строки выглядит следующим образом:

      К10А0А01А0А0001АД,

      и поэтому AAbB сводится к этому:

      C10A0AA01A0AA0001AAD.

      Нет b, который делал бы это несовместимым, поэтому всегда есть решение. Наоборот, если A не имеет стержня в каждой строке, то его уменьшенная ступенчатая форма строки выглядит так:

      К10А0А01А0А00000Д,

      , что может привести к противоречивой системе после добавления b:

      C10A0A001A0A00000016D.

      Напомним, что эквивалентно означает, что для любой заданной матрицы A либо все условий приведенной выше теоремы верны или все они ложны.

      Будьте внимательны при чтении утверждения приведенной выше теоремы. Первые два условия очень похожи на это примечание, но логически они совершенно разные из-за квантификатора « для всех b».

      Интерактив: критерии теоремы выполнены
      Интерактив: Критерии теоремы не выполняются

      Комментарии, исправления или предложения? (Требуется бесплатная учетная запись GitHub)

      Решение систем линейных уравнений с матрицами

      Правовые операции с матрицами

      Рис. 1

      На Рис. решение системы уравнений черным шрифтом слева. Этот система была решена путем сложения двух уравнений. Это дополнение вызвало переменная y отбрасывается, и получается уравнение с одной переменной. Этот позволили нам решить на х . Как только мы узнали x , мы могли легко найти и .

      Справа на рис. 1 выше, в синяя надпись , вы можете видеть аналогичную работу над строками матрицы. Строка матрицы 1,1,8 просто представляет уравнение: 1 x + 1 y = 8. Таким образом, строка матрицы — это просто коэффициенты уравнения. Матрица ряд 1,-1,4 представляет уравнение: 
      1 х — 1 у = 4.

      Так же, как мы можем сложить два уравнения слева, мы можем добавлять к строкам справа. Вы можете видеть это в строке с пометкой a + b —> a , где вы видите новый ряд 2,0,12. Это просто сумма строк a и b . Следующая строка помечена a b —> b показывает результат вычитания строки b из строки a , а именно, 0,2,4.

      Последний раздел справа показывает, что только как законно умножать уравнение на число или делить уравнение на число число, допустимо также умножать или делить строку матрицы на число (скаляр).

      В левом нижнем углу находится краткая информация о операции внутри матриц. Вы можете на законных основаниях выполнить любую из этих трех процедур. Вы также можете выполнять любую комбинацию этих трех операций. Действительно, эти юридические операции — это те же допустимые операции, которым вы научились ранее использовать в алгебре.

      Решение системы уравнений с помощью Алгебра

      Рисунок 2

      На рисунке 2 выше мы берем решение система сначала использует методы алгебры, которые вы должны знать из своей алгебры фон. Система трех уравнений показана в левом верхнем углу. Сначала я напомнить вам о том, что если вы решаете систему с тремя переменными, вы нужно 3 уравнения. Если вы решаете систему с 9 переменными, вам нужно 9уравнения. В общей ситуации, если вы решаете систему с n переменных, вам нужно n уравнений. Итак, вы видите, что для этой системы с 3 переменных, у нас есть 3 уравнения.

      В процессе решения такой системы мы нужно исключить переменные, пока мы не дойдем до уравнения с 1 переменной. Поэтому нам нужно найти какие-то способы легально избавиться от некоторых переменных.

      Вы можете заметить, что если добавить уравнения а и b , вы можете удалить переменную y . Вы также можете получить y отбросить, если вы добавите уравнения b и c .

      Теперь у нас осталось 2 уравнения с 2 переменные. Выше эти уравнения помечены как уравнения d и e . Нам еще нужно избавиться от еще одной переменной. Мы видим, что если умножить уравнение d на 2 и добавить его к уравнению e , мы действительно можем решить для одной из наших переменных. Мы находим, что x = 1. Это не займет много времени дольше, чтобы увидеть из уравнения b, что y должен быть равен -2. И наконец, подставляя x = 1 и y = -2 в уравнение a , мы можем найдите, что z должно равняться -1. Мы не закончим, пока не проверим все три приводит к каждому из трех уравнений, с которых мы начали.

      Красная стрелка на рисунке 2 указывает на матрицу форме тех же трех уравнений, с которых мы начали. Эта матрица 3 x 4 имеет квадрат матрица слева и дополнительный столбец справа. Поскольку эта матрица имеет был добавлен, его иногда называют расширенная матрица . В настоящее время мы будем работать над решением этой же системы, просто используя матрицу.

      Рисунок 3

      На рисунке 3 выше показаны два важных элемента. для нашей процедуры решения матрицы. В верхней части рисунка 3 вы видите «Желаемая цель». Это форма, которую мы хотим, чтобы наша матрица была по образцу. Обратите внимание, что мы хотим, чтобы левая часть матрицы стала тождеством . матрица . Также будет дополнительная колонка с тремя величинами в них. Эти величины a , b и c , причем решения системы. Верхний ряд гласит: «1 x + 0 y + 0 z = a «, что на самом деле говорит: « x = a «. вторая строка фактически говорит: « y = b ». Третья строка фактически говорит: « z = c ». Итак, матрица в таком виде полностью решается.

      Вторая часть рисунка 3 выше — это рекомендуемый стратегический порядок получения левой части матрицы преобразуется в единичную матрицу . Причина предоставления вам этот рекомендуемый стратегический порядок состоит в том, чтобы свести к минимуму или устранить впустую усилие. По мере того, как мы будем работать над проблемой ниже, вы в конце концов увидите, что это заказ поможет устранить любую напрасную работу.

      Рисунок 4

      На рисунке 4 выше мы видим, что 1 в второй ряд обведен. Это потому, что в нашем плане стратегического заказа мы в первую очередь преобразование этой первой ячейки во вторую строку. Что мы хотим этого первой ячейкой во втором ряду стать? Беглый взгляд на рисунок 3 должен сказать нам, что мы хотим, чтобы это был ноль. Как мы можем заставить это стать нулем? Что ж, мы, конечно, должны следовать только допустимым операциям, показанным на рис. 1. Один план, который позволит достичь этого, состоит в том, чтобы умножить строку b на -2 и добавить результат в строке a . Затем мы поместим результат в строку b . Этот обозначен выше с помощью обозначения -2 b + a —> b . Конечно, мы должны выполнить эту операцию для каждого числа в строке b . (все 4) Во второй матрице виден результат после переноса вне этой операции.

      Рисунок 5

      : Следующее число, которое нам нужно изменить, это нижний левый элемент, который обведен кружком на рис. 5 выше. Нам снова нужно это число, которое нужно преобразовать в ноль. Мы видим, что строка b не делает хороший. Ноль плюс что угодно ничего не меняет. Итак, теперь мы знаем, что у нас есть использовать строку a . Мы могли бы взять 3 строки и и -2 строки c . Сложение этих двух вместе даст ноль в первой позиции. Рисунок 5 показывает, что произойдет, если вы выполните этот план на весь ряд.

      Рисунок 6

      Следующее число, которое нам нужно изменить, это второй элемент в строке c . Нам нужно, чтобы этот элемент был нулем. Это очень важно на данном этапе задачи помнить, что пока мы хотим меняем второй элемент в строке на , нам нужно защитить ноль, который мы ранее было изменено на ноль. Для защиты нуля в первом позицию, нам нужно работать со строкой, которая также имеет ноль в первой должность. Это говорит нам о том, что нам нужно выполнить операцию, которая включает строку 9. 1327 б . Наш план состоит в том, чтобы умножить -3 на строку c , добавить к b и подставить c . Выполните эту операцию и посмотрите, получится ли у вас такая же нижняя строка, как показано на рисунке 6. выше.

      Рисунок 7

      Следующее число, которое нам нужно изменить, это второй элемент в строке b . Это первый элемент, который мы хотим превратиться в 1. Вам понравятся эти преобразования. Чтобы преобразовать element в 1, требуется только операция с одной строкой. Все, что вам нужно сделать, это разделить каждый элемент строки б по -3. Результаты вернутся в ряд б . См. результаты на рис. 7 выше.

      Рисунок 8

      Следующий элемент, который необходимо преобразовать является третьим элементом в строке c . Его нужно преобразовать в 1. Преобразования элемента в единицу являются самыми простыми из всех. Опять же, все мы нужно умножить каждый элемент в строке c на -1/20.

      Рисунок 9

      Наше следующее преобразование — третий элемент в ряд б . Этот элемент должен быть нулем. Здесь забота состоит в том, чтобы «защитить» первые два элемента этого ряда, над которыми мы работали трудно получить в их нынешнем виде. Два начальных нуля в строке c равны идеальная защита для этих двух элементов. Поэтому мы хотим сделать b,c операция. Наш план состоит в том, чтобы умножить строку c на 1/3, и прибавить результат к б .

      Рисунок 10

      Готово два ряда! Обратите внимание, что теперь строки b и c полностью готовы. Сейчас мы работаем над преобразованием третьего элемент в строке a до нуля. В ряду и нет ничего, что нуждается в защите. Мы просто вычтем строку из строки минус строку c , и поместите результат обратно в строку a .

      Рисунок 11

      Теперь нам нужно преобразовать второй элемент в строку и в ноль. У нас есть один элемент для защиты в строке и , Итак, мы замечаем, что строка b является идеальной защитой, потому что в ней стоит ноль. третья позиция. Здесь мы можем просто добавить строку a к строке b и поставить назад в ряду и .

      Рисунок 12

      Последний элемент для преобразования. Потому что это элемент должен быть единицей, мы можем просто разделить строку a на 2. Готово! Теперь, поскольку матрица имеет единичную матрицу слева, наши ответы таковы: сидит в третьей колонне. Система решена.

       

       

      Особые ситуации

      Некоторые системы уравнений не имеют решений. Если вы представляете система уравнений выше как система трех линий в трехмерном пространство. то вы вполне можете себе представить, что 3 таких линии не обязательно должны пересекаться в 1 общая точка. На самом деле, было бы довольно редко, чтобы они пересекались в одном месте. общая точка.

      При решении матриц мы изо всех сил пытаемся сделать левую часть матрицы в единичную матрицу. Если матрица не имеет решение, то вы увидите матрицы, которые в конечном итоге будут выглядеть так:



      Обратите внимание, что нижняя строка фактически говорит нам, что 0*X + 0*Y + 0*Z = 1, что невозможно. Это наша реплика, что это невозможно Матрица, из которой нужно найти единственное решение.

      В результате наших усилий по достижению единичная матрица в левой части матрицы. Ниже находится этот тип:


      Здесь у нас есть уникальный нижний ряд из всех нулей. Алгебраический уравнение, которое это представляет, а именно, 0*X + 0*Y + 0*Z = 0 верно. это правда, но это точно ничего не говорит нам о системе уравнения. Эта система имеет бесконечно много решений.

      Вы должны уметь распознавать эти два специальных типа результатов матрицы и что они означают.

      Системы линейных уравнений и матриц

      Марко Табога, доктор философии

      В этой лекции мы покажем, как матрицы и векторы могут использоваться для представления и анализа систем линейных уравнений.

      СОДЕРЖАНИЕ

      1. Системы линейных уравнений

      2. Растворы

      3. Репрезентация матрицы

        70033.

        70033.

        70033.

        .

      4. Решенные упражнения

        1. Упражнение 1

        2. Упражнение 2

      Системы линейных уравнений

    2. 3 Система линейные уравнения в неизвестные — это множество уравнениягде являются неизвестные и (за а также ) а также (за ) являются известными константами.

      Решения

      Неизвестные — это значения, которые мы хотели бы найти. Решение системы линейные уравнения означают нахождение набора значений для такое, что все уравнения выполняются. Такой набор называется решением система.

      Пример Определить систему Это это система 2 уравнений с 2 ​​неизвестными. Решение системы который можно проверить, подставив эти два значения в система:

      В общем, решение не гарантируется. Если он есть, то его нет гарантированно будет уникальным. Поэтому теория линейных уравнений касается трех основных аспектов:

      • вывод условий существования решений линейной системы;

      • понимание того, является ли решение уникальным, и как множественные решения связанные друг с другом;

      • нахождение методов, позволяющих находить решения линейной системы.

      Матричное представление системы

      Вышеупомянутая система линейные уравнения в неизвестные могут быть представлены компактно с использованием матриц в виде следует: где:

      • это вектор неизвестных ;

      • это матрица коэффициентов, -й элемент это константа, которая умножается в -й уравнение системы;

      • это вектор констант .

      Чтобы понять, как работает представление, обратите внимание, что это вектор, чей -й элемент равен точке продукт -й ряд а также , что г.,

      Следовательно,

      Пример Система Можно быть представленным как где в матрица коэффициентов это вектор неизвестных остров в вектор постоянных членов

      Наличие решений

      Записав систему линейных уравнений в матричной форме, мы можем легко получить общие условия существования решения.

      Предложение Линейная система имеет решение тогда и только тогда, когда принадлежит пролету столбцов из .

      Доказательство

      Продукт Можно интерпретировать как линейную комбинацию столбцов , с коэффициентами, взятыми из . Поэтому задача решения системы равносильна нахождению вектора коэффициентов что позволяет нам писать как линейная комбинация столбцов . Но может быть записана как линейная комбинация столбцов тогда и только тогда, когда он принадлежит их промежутку.

      Уникальность решения

      Приведем теперь общее условие единственности решения.

      Предложение Если линейная система имеет решение, то решение единственно тогда и только тогда, когда столбцы линейно независимый

      Доказательство

      Давайте сначала докажем часть if. У нас есть выше доказано, что решение существует тогда и только тогда, когда принадлежит пролету колонн . Если столбцы из линейно независимы, то они образуют основе их размаха. Кроме того, представление любого вектора размаха в виде линейного комбинация основы уникальна. Поэтому, если столбцы находятся линейно независимы, существует только одна их линейная комбинация, которая дает в результате, т. е. решение системы единственно. Давайте теперь докажем только если часть. Мы собираемся доказать, что если столбцы не независимыми, то существует более одного решения. Позволять быть решением, которое когда колонны линейно зависимы, существует ненулевой вектор что удовлетворяет как следствие, есть бесконечные решения, потому что является решением системы для любого скаляра :

      Несколько растворов

      Справедливо следующее предложение о кратных решениях.

      Предложение Если линейная система имеет решение и столбцы не являются линейно независимыми, то существуют бесконечные решения.

      Доказательство

      См. предыдущее доказательство.

      Решенные упражнения

      Ниже вы можете найти несколько упражнений с поясненными решениями.

      Упражнение 1

      Найдите матричное представление системы

      Решение

      Система может быть представлена как где в матрица коэффициентов это вектор неизвестных остров в вектор постоянных членов

      Упражнение 2

      ОпределитьЗапись вниз по уравнениям система

      Решение

      Два уравнения систем

      Как цитировать

      Пожалуйста, указывайте как:

      Taboga, Marco (2021). «Системы линейных уравнений и матрицы», Лекции по матричной алгебре. https://www.statlect.com/matrix-алгебра/системы-линейных-уравнений-и-матриц.

      Решение линейных систем с использованием матриц

      Содержание
      • Методы сокращения строк
      • Интерпретация решений из сокращенной формы эшелона строк
      • Умножение на обратную матрицу коэффициентов
      • Правило Крамера

      Систему уравнений можно представить в нескольких различных матричных формах. Один из способов состоит в том, чтобы реализовать систему как матричное произведение коэффициентов системы и вектор-столбца ее переменных. Квадратная матрица называется матрицей коэффициентов , потому что она состоит из коэффициентов переменных в системе уравнений:

      Произведение матриц: 2x+4y+7z=43x+3y+2z=85x+6y+3z=0 ⟶[247332563][xyz]=[480]. \text{Произведение матриц: } \quad \begin{array}{c c c c c c c} 2x & + & 4y & + & 7z & = & 4\\ 3x & + & 3y & + & 2z & = & 8\\ 5x&+&6y&+&3z&=&0\ \end{массив} \longrightarrow \left[ \begin{массив}{c c c} 2 и 4 и 7\\ 3 и 3 и 2 \\ 5 и 6 и 3 \\ \конец{массив}\справа] \left[ \begin{массив}{с} Икс \\ у \\ z\\ \end{массив} \right] = \left[ \begin{массив}{c} 4\\ 8\ 0 \end{массив} \right]. Произведение матриц: 2x3x5x​+++​4y3y6y​+++​7z2z3z​===​480​⟶⎣⎡​235​436​723​⎦⎤​⎣⎡​xyz​⎦⎤​=⎣⎡​480 ​⎦⎤​.

      Альтернативное представление, называемое расширенной матрицей , создается путем сшивания столбцов матриц вместе и разделения вертикальной чертой. Матрица коэффициентов помещается слева от этой вертикальной полосы, а константы в правой части каждого уравнения размещаются справа от вертикальной полосы:

      Расширенная матрица: 2x+4y+7z=43x+3y+ 2z=85x+6y+3z=0⟶[247433285630]. \text{Расширенная матрица: } \quad \quad \begin{array}{c c c c c c c} 2x & + & 4y & + & 7z & = & 4\\ 3x & + & 3y & + & 2z & = & 8\\ 5x&+&6y&+&3z&=&0\ \end{массив} \longrightarrow \left[ \begin{массив}{c c c | в} 2 и 4 и 7 и 4\\ 3 и 3 и 2 и 8\\ 5 и 6 и 3 и 0 \end{массив}\right]. Расширенная матрица: 2x3x5x​+++​4y3y6y​+++​7z2z3z​===​480​⟶⎣⎡​235​436​723​480​⎦⎤​​.

      Матрицами, представляющими эти системы, можно манипулировать таким образом, чтобы обеспечить легко читаемые решения. Эта манипуляция называется сокращением строки. Методы сокращения строк преобразуют матрицу в уменьшенную эшелонированную форму строк без изменения решений системы.

      Сокращенная ступенчатая форма строк матрицы AAA (\big((обозначается rref(A)) \text{rref}(A)\big)rref(A)) представляет собой матрицу равной размерности, которая удовлетворяет:

      1. Крайний левый ненулевой элемент в каждой строке равен 1 1 1. Этот элемент называется опорным.
      2. Любой столбец может иметь не более 1 1 1 точки поворота. Если у столбца есть точка опоры, то остальные элементы в столбце будут равны 0 0 0.
      3. Для любых двух столбцов C1C_{1} C1​ и C2C_{2}C2​, которые имеют сводные значения в строках R1 R_{1} R1​ и R2, R_{2}, R2​ соответственно, если сводные значения находятся в C1 C_{1 } C1​ находится слева от точки поворота в C2 C_{2}C2​, тогда R1 R_{1} R1​ выше R2 R_{2} R2​. Другими словами, для любых двух опорных точек P1 P_{1}P1​ и P2P_{2}P2​, если P2 P_{2}P2​ находится справа от P1 P_{1}P1​, то P2 P_{2} P2​ ниже P1 P_{1}P1​.
      4. Строки, состоящие только из нулей, находятся внизу матрицы.

      Чтобы преобразовать любую матрицу в ее уменьшенную ступенчатую форму строк, выполняется исключение Гаусса-Жордана. Существуют три элементарные операции со строками, используемые для получения формы уменьшенного эшелона строк:

      1. Переключить две строки.
      2. Умножить строку на любую ненулевую константу.
      3. Добавить скалярное число, кратное одной строке, к любой другой строке.

      Найдите rref(A) \text{rref}(A)rref(A) методом исключения Гаусса-Жордана, где

      A=[26−216−4−149]. A = \left[ \begin{массив}{c c c} 2 и 6 и -2\\ 1 и 6 и -4 \\ -1 и 4 и 9 \\ \end{массив}\right].A=⎣⎡​21−1​664​−2−49​⎦⎤​.


      Крайний левый элемент в первой строке должен быть равен 1, поэтому первая строка делится на 2:

      [26−216−4−149]→Разделить первую строку на 2.[13−116−4−149]. \left[ \begin{массив}{c c c} 2 и 6 и -2\\ 1 и 6 и -4 \\ -1 и 4 и 9 \\ \конец{массив}\справа] \ce{->[\large \text{Разделить первую строку на 2.}]} \left[ \begin{массив}{c c c} 1 и 3 и -1\\ 1 и 6 и -4 \\ -1 и 4 и 9\\ \end{массив}\right]. ⎣⎡​21−1​664​−2−49​⎦⎤​Разделите первую строку на 2.​⎣⎡​11−1 364​−1−49​⎦⎤​.

      Верхний левый элемент является опорным, поэтому остальные элементы в первом столбце должны быть равны 0. Это можно сделать, вычитая первую строку из второй строки. Кроме того, первую строку можно добавить к третьей строке, чтобы получить необходимые нули в первом столбце:

      .

      [13−116−4−149]→RX2−RX1 и RX3+RX1[13−103−3078]. \left[ \begin{массив}{c c c} 1 и 3 и -1\\ 1 и 6 и -4 \\ -1 и 4 и 9\\ \конец{массив}\справа] \ce{->[\large R_2 — R_1 \text{ и } R_3 + R_1]} \left[ \begin{массив}{c c c} 1 и 3 и -1\\ 0 и 3 и -3 \\ 0 и 7 и 8 \\ \end{массив}\right]. ⎣⎡​11−1​364​−1−49​⎦⎤​RX2​−RX1​ и  RX3​+RX1​​⎣⎡​100 337​−1−38​⎦⎤​.

      Теперь, когда крайний левый столбец равен [100] \left[ \begin{array}{c} 1\\ 0 \\ 0 \\ \end{array}\right] ⎣⎡​100​⎦⎤​, средний элемент можно сделать равным 1, разделив вторую строку на 3:

      [13−103−3078]→ Разделить вторую строку на 3 [13−101−1078]. \left[ \begin{массив}{c c c} 1 и 3 и -1\\ 0 и 3 и -3 \\ 0 и 7 и 8 \\ \конец{массив}\справа] \ce{->[\large \text{Поделить вторую строку на 3.}]} \left[ \begin{массив}{c c c} 1 и 3 и -1\\ 0 и 1 и -1 \\ 0 и 7 и 8 \\ \end{массив}\right]. ⎣⎡​100​337​−1−38​⎦⎤​Поделите вторую строку на 3. ​⎣⎡​100 317​−1−18​⎦⎤​.

      Верхний и нижний элементы второго столбца можно сделать равными 0 с помощью соответствующих операций над строками:

      [13−101−1078]→RX1−3 RX2 и RX3−7 RX2[10201−10015]. \left[ \begin{массив}{c c c} 1 и 3 и -1\\ 0 и 1 и -1 \\ 0 и 7 и 8 \\ \конец{массив}\справа] \ce{->[\large R_1 — 3R_2 \text{ и } R_3 — 7R_2]} \left[ \begin{массив}{c c c} 1 и 0 и 2\\ 0 и 1 и -1 \\ 0 и 0 и 15 \\ \end{массив}\right]. ⎣⎡​100​317​−1−18​⎦⎤​RX1​−3RX2​ и  RX3​−7RX2​​⎣⎡​100 010​2−115​⎦⎤​.

      Теперь средний столбец [010] \left[ \begin{array}{c} 0 \\ 1\\ 0 \\ \end{array}\right] ⎣⎡​010​⎦⎤​, метод переходит к третьему столбцу путем деления третьей строки на 15:

      [10201−10015]→ Разделить третью строку на 15[10201−1001]. \left[ \begin{массив}{c c c} 1 и 0 и 2\\ 0 и 1 и -1 \\ 0 и 0 и 15 \\ \конец{массив}\справа] \ce{->[\large \text{Разделить третью строку на 15} ]} \left[ \begin{массив}{c c c} 1 и 0 и 2\\ 0 и 1 и -1 \\ 0 и 0 и 1 \\ \end{массив}\right]. ⎣⎡​100​010​2−115​⎦⎤​Разделите третью строку на 15​⎣⎡​100 010​2−11​⎦⎤​.

      На последнем этапе процесса к первой и второй строкам добавляются числа, кратные третьей строке, так что последний столбец становится [001]: \left[ \begin{array}{c} 0 \\ 0 \\ 1\\ \end{массив}\right]: ⎣⎡​001​⎦⎤​:

      [10201−1001]→RX1−2 RX3 и RX2+RX3[100010001]. □ \left[ \begin{массив}{c c c} 1 и 0 и 2\\ 0 и 1 и -1 \\ 0 и 0 и 1 \\ \конец{массив}\справа] \ce{->[\large R_1 — 2R_3 \text{ и } R_2 + R_3 ]} \left[ \begin{массив}{c c c} 1 и 0 и 0\\ 0 и 1 и 0 \\ 0 и 0 и 1 \\ \end{массив}\right]. \ _\площадь ⎣⎡​100​010​2−11​⎦⎤​RX1​−2RX3​  и  RX2​+RX3​​​⎣⎡​100 010 001​⎦⎤​. □​

      A = [14−564022−1] A = \left[ \begin{array}{ccc} 1 и 4 и -5 \\ 6 и 4 и 0 \\ 2&2&-1\ \end{array} \right] A=⎣⎡​162​442​−50−1​⎦⎤​

      Какова сумма всех элементов в rref(A)? \text{rref}(A)?rref(A)?

      Примечание : rref(A)\text{rref}(A) rref(A) обозначает «эшелонированную форму сокращенной строки» матрицы A.A.A.


      Для получения решений системы линейных уравнений можно использовать следующие шаги:

      1. Преобразование данных уравнений в расширенную матрицу.
      2. Выполните операции со строками, чтобы получить сокращенную ступенчатую форму матрицы строк.
      3. Преобразование в расширенную матрицу обратно в набор уравнений.

      В этой форме возможные решения системы линейных уравнений, которые представляет расширенная матрица, могут быть определены тремя случаями.

      Случай 1. Если rref(A)\text{rref}(A)rref(A) — единичная матрица, то система имеет единственное решение.

      При чтении построчно эта расширенная матрица говорит: x=−1,y=2,x=-1, y=2,x=-1,y=2 и z=3:z=3:z=3 :

      [100−101020013]. \left[ \begin{массив}{c c c | в} 1 и 0 и 0 и -1\\ 0 и 1 и 0 и 2\\ 0 и 0 и 1 и 3\\ \end{массив}\right]. ⎣⎡​100​010​001​−123​⎦⎤​.

      Случай 2. Если rref(A)\text{rref}(A)rref(A) содержит строку нулей, за которой следует увеличенное нулевое значение, то система имеет бесконечно много решений.

      Рассмотрим следующую расширенную матрицу в сокращенной эшелонированной форме строк. Последняя строка читается как [000∣0]\left[0 \hspace{0,15cm} 0 \hspace{0,15cm} 0 \hspace{0,15cm} | \hspace{.15cm} 0\right] [000∣0] или 0=0,0 = 0,0=0, что не является противоречием. Остальные строки читаются как x+4y=-1x + 4y = -1x+4y=-1 и z=2.z = 2.z=2. Это определяет значение для z,z,z, но существует бесконечно много пар действительных чисел xxx и yyy таких, что x+4y=-1,x+4y = -1,x+4y=-1, так что эта система имеет бесконечное количество решений:

      [140−100120000]. \left[ \begin{массив}{c c c | в} 1 и 4 и 0 и -1\\ 0 и 0 и 1 и 2\\ 0 и 0 и 0 и 0\\ \end{массив}\right]. ⎣⎡​100​400​010​−120​⎦⎤​.

      Случай 3. Если rref(A)\text{rref}(A)rref(A) содержит строку нулей, за которой следует ненулевое увеличенное значение, то система не имеет решения.

      Этот случай похож на первый тем, что нижняя строка содержит в основном 0. Однако обратите внимание, что последняя строка матрицы имеет вид [000∣1]\left[0 \hspace{0,15cm} 0 \hspace{0,15cm} 0 \hspace{0,15cm} | \hspace{. 15cm} 1\right] [000∣1], что читается как 0=1.0 = 1.0=1. Это противоречие; таким образом, система не имеет решений:

      [140-100120001]. \left[ \begin{массив}{c c c | в} 1 и 4 и 0 и -1\\ 0 и 0 и 1 и 2\\ 0 и 0 и 0 и 1\\ \end{массив}\right]. ⎣⎡​100​400​010​−121​⎦⎤​.

      Найдите значения x x x и y y y, удовлетворяющие следующей системе уравнений:

      {5x−y=1x+2y=9. \begin{случаи} 5х — у =1\\ х + 2у = 9. \end{cases} {5x−y=1x+2y=9.​


      В этом примере предпочтение отдается методам сокращения строк, а не стандартному исключению. Данную систему уравнений можно записать в виде следующей расширенной матрицы:

      [5−11129]. \left[ \begin{массив} {cc|c} 5 и -1 и 1\\ 1 и 2 и 9\\ \end{массив} \right]. [51​−12​19​].

      Исключение Гаусса-Жордана выполняется для получения сокращенной ступенчатой ​​формы матрицы строк:

      [5−11129] → Поменять местами RX1 и RX2 ⋅ [1295−11] \left[ \begin{array} {cc|c} 5 и -1 и 1\\ 1 и 2 и 9\\ \end{массив} \right] \ce{->[\text{Поменять местами } R_1 \text{ и } R_2. ]} \left[ \begin{массив} {cc|c} 1 и 2 и 9\\ 5 и -1 и 1\\ \end{массив} \right] [51​−12​19​]Поменять местами  RX1​  и  RX2​⋅​[15​2−1​91​]

      [1295−11]→RX2−5 RX1[1290−11−44] \left[ \begin{array} {cc|c} 1 и 2 и 9\\ 5 и -1 и 1\\ \end{массив} \right] \ce{->[R_2 — 5R_1 ]} \left[ \begin{массив} {cc|c} 1 и 2 и 9\\ 0 и -11 и -44\\ \end{массив} \right] [15​2−1​91​]RX2​−5RX1​[10​2−11​9−44​]

      [1290−11−44]→ Разделить вторую строку на −11 ⋅ [129014] \left[ \begin{array} {cc|c} 1 и 2 и 9\\ 0 и -11 и -44\\ \end{массив} \right] \ce{->[\text{Разделить вторую строку на } -11.]} \left[ \begin{массив} {cc|c} 1 и 2 и 9\\ 0 и 1 и 4\\ \end{array} \right] [10​2−11​9−44​]Разделить вторую строку на -11⋅​[10​21​94​]

      [129014]→RX1−2 RX2[101014]. \left[ \begin{массив} {cc|c} 1 и 2 и 9\\ 0 и 1 и 4\\ \end{массив} \right] \ce{->[R_1 — 2R_2]} \left[ \begin{массив} {cc|c} 1 и 0 и 1\\ 0 и 1 и 4\\ \end{массив} \right]. [10​21​94​]RX1​−2RX2​[10​01​14​].

      Первая строка этой сокращенной формы читается как x=1 x = 1 x=1, а вторая строка читается как y=4 y = 4 y=4. {-1}A−1: 9{-1}б. Ах=b⟹A−1Ax=A−1b⟹x=A−1b.

      Одна ошибка этого метода возникает, когда det⁡(A)=0.\det(A) = 0.det(A)=0. Поскольку определитель AAA равен 0, он не может быть обратным, а это означает, что этот метод не работает. Фактически, det⁡(A)=0\det(A) = 0det(A)=0 эквивалентно rref(A)\text{rref}(A)rref(A), содержащему строку нулей. Таким образом, система уравнений, которую представляет Ax=bAx = bAx=b, не будет иметь единственного решения. Однако система может иметь бесконечно много решений или вообще не иметь решений, и для ее обнаружения потребуется другой метод.

      -3 0 3 9

      Система уравнений задается как

      ax+by=5cx+dy=−1.\begin{aligned} ax + by &= 5 \\ сх + dy &= -1. \end{aligned} ax+bycx+dy​=5=−1.​

      Если инверсия [abcd]\left[ \begin{array}{cc} а и б \\ c & d \end{array} \right][ac​bd​] равно [32−31]\left[ \begin{array}{cc} 3 и 2 \\ -3 & 1 \end{array} \right][3−3​21​], тогда что такое x+y?x + y?x+y? 92 = -1i2=−1, а ∣z∣|z|∣z∣ — абсолютное значение zzz.

      Правило Крамера — это формула, использующая определители для решения системы линейных уравнений. В приведенном ниже утверждении правила используются только три переменные, но правило можно применять к системе любого размера.

      Рассмотрим систему уравнений с тремя переменными:

      a1x+b1y+c1z=d1a2x+b2y+c2z=d2a3x+b3y+c3z=d3.\begin{aligned} a_{1} x + b_{1} y + c_{1} z &= d_{1} \\ a_{2} x + b_{2} y + c_{2} z &= d_{2} \\ a_{3} x + b_{3} y + c_{3} z &= d_{3}. \end{align}a1​x+b1​y+c1​za2​x+b2​y+c2​za3​x+b3​y+c3​z​=d1​=d2​=d3​.​

      Решение этой системы:

      x=det⁡(A1)det⁡(A)y=det⁡(A2)det⁡(A)z=det⁡(A3)det⁡(A),x =\dfrac{\det(A_{1})}{\det(A)} \quad y = \dfrac{\det(A_{2})}{\det(A)} \quad z= \dfrac{ \det(A_{3})}{\det(A)},x=det(A)det(A1​)​y=det(A)det(A2​)​z=det(A)det(A3 ​)​,

      , где A=[a1b1c1a2b2c2a3b3c3]A= \begin{bmatrix} a_{1} & b_{1} & c_{1} \\ a_{2} & b_{2} & c_{2} \ \ a_{3} & b_{3} & c_{3} \end{bmatrix} A=⎣⎡​a1​a2​a3​b1​b2​b3​​c1​c2​c3​​⎦⎤​ матрица коэффициентов и каждый AiA_{i}Ai​ представляет собой AAA с заменой столбца ihi^\text{th} на [d1d2d3]. \begin{bmatrix} d_{1} \\ d_{2} \\ d_{3} \end{bmatrix}.⎣⎡​d1​d2​d3​⎦⎤​.

      Примечание:

      • det⁡(A2),\det(A_2),det(A2​) или det⁡(A3) \det(A_3)det(A3​) не равно 000, то система не имеет решения.
      • Если det⁡(A)=0\det(A) = 0det(A)=0 и det⁡(A1)\det(A_1)det(A1​), det⁡(A2),\det(A_2), det(A2​) и det⁡(A3) \det(A_3)det(A3​) равны 0, то решение может существовать, а может и не существовать. Если решение существует, то система уравнений будет иметь бесконечно много решений.
      • Вычислительная сложность этого метода увеличивается на порядок O(n!),O(n!),O(n!), поэтому он становится громоздким после 3×33\times 3 3×3 систем.

      Как указывалось ранее, система уравнений может быть представлена ​​как произведение матриц типа

      .

      [a1b1c1a2b2c2a3b3c3][xyz]=[d1d2d3].\begin{bmatrix} a_{1} & b_{1} & c_{1} \\ a_{2} & b_{2} & c_{2} \\ a_ {3} & b_{3} & c_{3} \end{bmatrix}\begin{bmatrix} x \\ y \\ z \end{bmatrix} = \begin{bmatrix} d_{1} \\ d_{2 } \\ d_{3} \end{bmatrix}. ⎣⎡​a1​a2​a3​​b1​b2​b3​​c1​c2​c3​​⎦⎤​⎣⎡​xyz​⎦⎤​=⎣ ⎡​d1​d2​d3​​⎦⎤​.

      Определитель матрицы коэффициентов AAA равен

      .

      det⁡(A)=∣a1b1c1a2b2c2a3b3c3∣.\det(A) = \left| \begin{array}{ccc} a_{1} & b_{1} & c_{1} \\ a_{2} & b_{2} & c_{2} \\ a_{3} & b_{3} & c_{3} \end{массив} \right|.det(A)=∣∣∣∣∣∣​a1​a2​a3​​b1​b2​b3​​c1​c2​c3​​∣∣∣∣ ∣∣​.

      Если столбец или строка матрицы умножается на константу, то значение ее определителя умножается на эту константу. Таким образом, при умножении первого столбца на x,x,x это уравнение становится равным 9.0034

      xdet⁡(A)=∣a1xb1c1a2xb2c2a3xb3c3∣.x\det(A) = \left| \begin{array}{ccc} a_{1}x & b_{1} & c_{1} \\ a_{2}x & b_{2} & c_{2} \\ a_{3}x & b_{ 3} & c_{3} \end{массив} \right|.xdet(A)=∣∣∣∣∣∣​a1​xa2​xa3​x​b1​b2​b3​​c1​c2​c3​​ ∣∣∣∣∣∣​.

      Здесь стоит отметить, что методы сокращения столбцов можно использовать так же, как если бы они были методами сокращения строк, сохраняя при этом решение. Таким образом, добавление yyy, умноженное на второй столбец, и zzz, умноженное на третий столбец, к первому столбцу дает 9. 0034

      xdet⁡(A)=∣a1xb1c1a2xb2c2a3xb3c3∣→C1+yC2+zC3∣a1x+b1y+c1zb1c1a2x+b2y+c2zb2c2a3x+b3y+c3zb3c3∣.x\det(A) = \left| \begin{array}{ccc} a_{1}x & b_{1} & c_{1} \\ a_{2}x & b_{2} & c_{2} \\ a_{3}x & b_{ 3} & c_{3} \end{массив} \right| \xrightarrow[]{C_{1} + yC_{2} + zC_{3}} \left| \begin{array}{ccc} a_{1}x+b_{1}y+c_{1}z & b_{1} & c_{1} \\ a_{2}x+b_{2}y+c_ {2}z & b_{2} & c_{2} \\ a_{3}x+b_{3}y+c_{3}z & b_{3} & c_{3} \end{массив} \right |. xdet(A)=∣∣∣∣∣∣​a1​xa2​xa3​x​b1​b2​b3​​c1​c2​c3​​∣∣∣∣∣∣​C1​+yC2​+zC3​​ ∣∣∣∣∣∣​a1​x+b1​y+c1​za2​x+b2​y+c2​za3​x+b3​y+c3​z​b1​b2​b3​​c1​c2​ c3​​∣∣∣∣∣∣​.

      Однако, поскольку a1x+b1y+c1z=d1a_{1}x + b_{1}y + c_{1}z = d_1a1​x+b1​y+c1​z=d1​ (((и аналогично для d2d_2d2​ и d3),d_3),d3​), новый первый столбец в точности равен ∣d1d2d3∣. \влево| \begin{массив}{c} d_{1} \\ d_{2} \\ d_{3} \end{массив} \right|.∣∣∣∣∣∣​d1​d2​d3​​∣∣∣ ∣∣∣​. То есть

      xdet⁡(A)=∣a1x+b1y+c1zb1c1a2x+b2y+c2zb2c2a3x+b3y+c3zb3c3∣=∣d1b1c1d2b2c2d3b3c3∣=det⁡(A1) или xdet⁡(A)=det⁡x(A1)⟨ A1)det⁡(A). \begin{выровнено} х\det(A) = \влево| \begin{array}{ccc} a_{1}x+b_{1}y+c_{1}z & b_{1} & c_{1} \\ a_{2}x+b_{2}y+c_ {2}z & b_{2} & c_{2} \\ a_{3}x+b_{3}y+c_{3}z & b_{3} & c_{3} \end{массив} \right | = \ влево | \begin{array}{ccc} d_{1} & b_{1} & c_{1} \\ d_{2} & b_{2} & c_{2} \\ d_{3} & b_{3} & c_{3} \end{массив} \right| &= \det(A_1)\quad \text{ или } \\\\ х \ det (A) & = \ det (A_1) \ longrightarrow x = \ frac {\ det (A_1)} {\ det (A)}. \end{выровнено} xdet(A)=∣∣∣∣∣∣​a1​x+b1​y+c1​za2​x+b2​y+c2​za3​x+b3​y+c3​z​b1 ​b2​b3​​c1​c2​c3​∣∣∣∣∣∣​=∣∣∣∣∣∣​d1​d2​d3​​b1​b2​b3​​c1​c2​c3​​∣ ∣∣∣∣∣​xdet(A)​=det(A1​) или =det(A1​)⟶x=det(A)det(A1​)​.​

      Аналогичной конструкции,

      y=det⁡(A2)det⁡(A) и z=det⁡(A3)det⁡(A). □y = \ frac {\ det (A_2)} {\ det (A)} \ quad \ text {и} \ quad z = \ frac {\ det (A_3)} {\ det (A)}. \ _\squarey=det(A)det(A2​)​ и z=det(A)det(A3​)​. □​

      {5x−y=1x+2y=9 \begin{case} 5х — у =1\\ х + 2у = 9\\ \end{case} {5x−y=1x+2y=9​

      Согласно правилу Крамера, решение приведенной выше системы равно

      .
    3. Добавить комментарий

      Ваш адрес email не будет опубликован. Обязательные поля помечены *